SBA MCQ Flashcards

1
Q

A 25-year-old man presents to the emergency department with a stab wound to the left chest. He is tachycardic, hypotensive and has decreased breath sounds on the left side. What is the most appropriate initial management?
A) Insert a chest drain in the fifth intercostal space in the mid-axillary line
B) Insert a chest drain in the second intercostal space in the mid-clavicular line C) Perform a needle decompression in the second intercostal space in the mid-clavicular line
D) Perform a needle decompression in the fifth intercostal space in the mid-axillary line
E) Perform a pericardiocentesis

A

The correct answer is C. This patient has a tension pneumothorax, which is a life-threatening condition that requires immediate decompression of the pleural cavity. A needle decompression in the second intercostal space in the mid-clavicular line is the preferred method of initial management, as it is quick and effective. A chest drain can be inserted afterwards to prevent recurrence. A chest drain in the fifth intercostal space in the mid-axillary line is the standard treatment for a simple pneumothorax, but it is not suitable for a tension pneumothorax. A chest drain in the second intercostal space in the mid-clavicular line is not recommended, as it may cause injury to the internal mammary artery or the lung apex. A needle decompression in the fifth intercostal space in the mid-axillary line is an alternative site for decompression, but it is less reliable and more difficult to access. A pericardiocentesis is indicated for a cardiac tamponade, which is a different condition that may also result from a penetrating chest injury.

How well did you know this?
1
Not at all
2
3
4
5
Perfectly
2
Q

A 45-year-old woman undergoes a total thyroidectomy for a multinodular goitre. On the first postoperative day, she complains of tingling sensations in her fingers and toes. She also has muscle cramps and twitching of her facial muscles. What is the most likely diagnosis?
A) Hypocalcaemia
B) Hypokalaemia
C) Hypomagnesaemia
D) Hypophosphataemia
E) Hypothyroidism

A

The correct answer is A. This patient has symptoms of hypocalcaemia, which is a common complication of total thyroidectomy, due to inadvertent damage or removal of the parathyroid glands1. Hypocalcaemia can cause neuromuscular irritability, such as paraesthesia, tetany, Chvostek’s sign (facial muscle twitching elicited by tapping the facial nerve), and Trousseau’s sign (carpopedal spasm induced by inflating a blood pressure cuff above the systolic pressure)2. Hypocalcaemia is treated with oral or intravenous calcium supplementation, and sometimes vitamin D or calcitriol3. The other options are less likely to cause the patient’s symptoms, and can be ruled out by measuring the serum electrolyte levels and thyroid function tests.

How well did you know this?
1
Not at all
2
3
4
5
Perfectly
3
Q

A 65-year-old man undergoes a right hemicolectomy for colon cancer. On the third postoperative day, he develops fever, tachycardia, and abdominal pain. On examination, he has rebound tenderness and guarding in the right lower quadrant. His white blood cell count is 15 x 109/L and his C-reactive protein is 120 mg/L. What is the most likely diagnosis?
A) Anastomotic leak
B) Bowel obstruction
C) Incisional hernia
D) Surgical site infection
E) Urinary tract infection

A

The correct answer is A. This patient has signs of peritonitis, which is a serious complication of bowel surgery. The most common cause of peritonitis after a hemicolectomy is an anastomotic leak, which occurs when the connection between the two ends of the bowel breaks down1. This allows the bowel contents to spill into the peritoneal cavity, causing inflammation, infection, and sepsis2. The diagnosis of an anastomotic leak can be confirmed by a contrast enema or a CT scan3. The treatment involves drainage of the abscess, antibiotics, and sometimes reoperation. The other options are less likely to cause the patient’s symptoms, and can be ruled out by further investigations, such as abdominal X-ray, ultrasound, or urine culture.

How well did you know this?
1
Not at all
2
3
4
5
Perfectly
4
Q

A 55-year-old man presents with a two-month history of progressive dysphagia, weight loss, and retrosternal pain. He has a history of gastro-oesophageal reflux disease and Barrett’s oesophagus. An endoscopy reveals a 4 cm ulcerated mass in the lower third of the oesophagus. A biopsy confirms adenocarcinoma of the oesophagus. A CT scan shows no evidence of distant metastasis. What is the most appropriate management?
A) Chemoradiotherapy followed by surgery
B) Chemotherapy alone
C) Endoscopic mucosal resection
D) Palliative stent insertion
E) Surgery alone

A

The correct answer is A. This patient has a locally advanced adenocarcinoma of the oesophagus, which is associated with Barrett’s oesophagus1. The treatment of choice for this stage of disease is chemoradiotherapy followed by surgery, which has been shown to improve survival and quality of life compared to surgery alone2. Chemotherapy alone is not curative and is usually reserved for metastatic disease. Endoscopic mucosal resection is only suitable for early-stage tumours that are confined to the mucosa. Palliative stent insertion is indicated for patients who are unfit for surgery or have unresectable disease, to relieve dysphagia and improve nutrition. Surgery alone is not recommended, as it has a high morbidity and mortality rate and does not address the risk of local recurrence or distant spread3.

How well did you know this?
1
Not at all
2
3
4
5
Perfectly
5
Q

A 70-year-old man presents with a six-month history of progressive dyspnoea, orthopnoea, and peripheral oedema. He has a history of hypertension, diabetes, and coronary artery disease. On examination, he has a raised jugular venous pressure, a displaced apex beat, a pansystolic murmur at the apex, and bilateral crackles in the lungs. His chest X-ray shows cardiomegaly and pulmonary congestion. His electrocardiogram shows left ventricular hypertrophy and atrial fibrillation. His echocardiogram shows a dilated left ventricle with an ejection fraction of 30%. What is the most likely diagnosis?
A) Aortic stenosis
B) Aortic regurgitation
C) Mitral stenosis
D) Mitral regurgitation
E) Tricuspid regurgitation

A

The correct answer is D. This patient has signs and symptoms of heart failure, which is a condition where the heart is unable to pump enough blood to meet the body’s needs1. The most common cause of heart failure is ischaemic heart disease, which can damage the heart muscle and impair its contractility2. This patient has a history of coronary artery disease, which increases his risk of developing heart failure. The most likely valve lesion that is associated with ischaemic heart disease and heart failure is mitral regurgitation, which occurs when the mitral valve does not close properly and allows blood to leak back into the left atrium during systole3. This causes a volume overload on the left ventricle, leading to dilation, hypertrophy, and reduced ejection fraction. Mitral regurgitation also causes a pansystolic murmur at the apex, which is best heard with the bell of the stethoscope. The other options are less likely to cause the patient’s presentation, and can be ruled out by the echocardiogram findings. Aortic stenosis causes a pressure overload on the left ventricle, leading to concentric hypertrophy and a reduced stroke volume. It also causes a systolic ejection murmur at the right second intercostal space. Aortic regurgitation causes a volume overload on the left ventricle, leading to dilation and increased stroke volume. It also causes a diastolic decrescendo murmur at the left third intercostal space. Mitral stenosis causes a pressure overload on the left atrium, leading to dilation and atrial fibrillation. It also causes a diastolic rumbling murmur at the apex. Tricuspid regurgitation causes a volume overload on the right ventricle, leading to dilation and reduced ejection fraction. It also causes a pansystolic murmur at the left lower sternal edge.

How well did you know this?
1
Not at all
2
3
4
5
Perfectly
6
Q

A 35-year-old woman presents with a three-month history of a painless lump in her right breast. She has no family history of breast cancer. On examination, she has a 2 cm firm, mobile, and well-defined mass in the upper outer quadrant of her right breast. There is no nipple discharge, skin changes, or axillary lymphadenopathy. She undergoes a core needle biopsy of the mass, which reveals a benign fibroepithelial lesion. What is the most likely diagnosis?
A) Fibroadenoma
B) Phyllodes tumour
C) Intraductal papilloma
D) Fibrocystic change
E) Ductal carcinoma in situ

A

The correct answer is A. This patient has a fibroadenoma, which is the most common benign breast tumour in young women1. It is composed of fibrous and glandular tissue, and it is usually well-circumscribed, rubbery, and mobile2. It does not increase the risk of breast cancer, and it may regress spontaneously or with hormonal changes3. The treatment options include observation, excision, or percutaneous ablation. The other options are less likely to cause the patient’s presentation, and can be ruled out by the biopsy findings. A phyllodes tumour is a rare fibroepithelial lesion that can be benign or malignant, and it usually grows rapidly and forms a large mass. An intraductal papilloma is a benign proliferation of epithelial cells within a duct, and it usually presents with nipple discharge. Fibrocystic change is a common condition that causes breast pain, tenderness, and cysts, especially in premenopausal women. Ductal carcinoma in situ is a non-invasive form of breast cancer that originates from the ductal epithelium, and it usually presents as microcalcifications on mammography.

How well did you know this?
1
Not at all
2
3
4
5
Perfectly
7
Q

A 50-year-old man presents with a two-week history of a painless, hard, and fixed lump in his left testis. He has no other symptoms or risk factors. He undergoes an ultrasound scan of the scrotum, which confirms a solid mass within the left testis. A serum tumour marker panel shows elevated levels of alpha-fetoprotein (AFP) and human chorionic gonadotropin (hCG). What is the most likely diagnosis?
A) Seminoma
B) Teratoma
C) Leydig cell tumour
D) Sertoli cell tumour
E) Lymphoma

A

The correct answer is B. This patient has a teratoma, which is a type of germ cell tumour that can arise from the testis. It is composed of various types of tissue, such as hair, teeth, bone, and cartilage. It can secrete alpha-fetoprotein (AFP) and human chorionic gonadotropin (hCG), which are useful tumour markers for diagnosis and monitoring. The treatment of choice is radical orchiectomy, which involves removal of the entire testis and spermatic cord. The other options are less likely to cause the patient’s presentation, and can be ruled out by the ultrasound and tumour marker findings. A seminoma is a type of germ cell tumour that is usually pure and does not secrete AFP or hCG. A Leydig cell tumour is a type of sex cord-stromal tumour that can secrete testosterone and cause gynaecomastia. A Sertoli cell tumour is another type of sex cord-stromal tumour that is usually benign and does not secrete any hormones. A lymphoma is a type of haematological malignancy that can affect the testis, especially in older men, and can cause systemic symptoms such as fever, night sweats, and weight loss.

How well did you know this?
1
Not at all
2
3
4
5
Perfectly
8
Q

A 40-year-old woman presents with a one-week history of a painful, red, and swollen right leg. She has no history of trauma, surgery, or immobilisation. She is otherwise healthy and takes no medications. On examination, she has a temperature of 38.5°C, a pulse of 110 beats per minute, and a blood pressure of 150/90 mmHg. Her right leg is warm, erythematous, and tender, with a circumference of 45 cm at the mid-thigh level, compared to 40 cm on the left side. There is no evidence of skin breakdown, ulceration, or lymphangitis. A Doppler ultrasound scan of the right leg shows a thrombus in the femoral vein. A blood test shows a white blood cell count of 12 x 109/L, a haemoglobin of 14 g/dL, a platelet count of 300 x 109/L, and an erythrocyte sedimentation rate of 60 mm/h. What is the most likely diagnosis?
A) Cellulitis B) Deep vein thrombosis C) Lymphoedema D) Necrotising fasciitis E) Superficial thrombophlebitis

A

The correct answer is B. This patient has a deep vein thrombosis (DVT), which is a condition where a blood clot forms in a deep vein, usually in the lower limbs. It can cause pain, swelling, redness, and warmth in the affected leg, and it can also lead to serious complications such as pulmonary embolism and post-thrombotic syndrome. The risk factors for DVT include trauma, surgery, immobilisation, cancer, pregnancy, oral contraceptives, smoking, obesity, and inherited or acquired thrombophilia. The diagnosis of DVT can be confirmed by a Doppler ultrasound scan, which shows the presence and location of the thrombus. The treatment of DVT involves anticoagulation therapy, such as heparin or warfarin, to prevent the clot from growing or breaking off. The other options are less likely to cause the patient’s presentation, and can be ruled out by the clinical and radiological findings. Cellulitis is a bacterial infection of the skin and subcutaneous tissue, which causes erythema, warmth, tenderness, and fever, but it usually affects only one area of the leg and is associated with skin breakdown, ulceration, or lymphangitis. Lymphoedema is a chronic swelling of the limb due to impaired lymphatic drainage, which causes a non-pitting oedema, skin thickening, and fibrosis, but it is usually painless and bilateral. Necrotising fasciitis is a rare and severe infection of the fascia and subcutaneous tissue, which causes severe pain, fever, crepitus, and systemic toxicity, but it is usually preceded by trauma, surgery, or immunosuppression. Superficial thrombophlebitis is an inflammation of a superficial vein, which causes a palpable, tender, and erythematous cord along the course of the vein, but it is usually mild and self-limiting.

How well did you know this?
1
Not at all
2
3
4
5
Perfectly
9
Q

A 74yo man who has been a smoker since he was 20 has recently been dx with SCLC. What serum electrolyte picture will confirm the presence of SIADH?
a. High serum Na, low serum osmolarity, high urine osmolarity
b. Low serum Na, high serum osmolarity, high urine osmolarity
c. High serum Na, low serum osmolarity, low urine osmolarity
d. Low serum Na, low serum osmolarity, high urine osmolarity
e. High serum Na, high serum osmolarity, low urine osmolarity

A

The correct answer is:

b. Low serum Na, high serum osmolarity, high urine osmolarity

This electrolyte profile is characteristic of Syndrome of Inappropriate Antidiuretic Hormone (SIADH) secretion, which can occur in patients with small cell lung cancer (SCLC) due to ectopic production of antidiuretic hormone (ADH) by the tumor cells. The low serum sodium (Na) level, combined with high serum osmolarity and high urine osmolarity due to water retention, reflects the dilutional hyponatremia typical of SIADH. This electrolyte imbalance is often seen in patients with SCLC, particularly in those with advanced disease. It’s crucial to recognize and manage SIADH promptly to prevent potentially serious complications.

How well did you know this?
1
Not at all
2
3
4
5
Perfectly
10
Q

A 67yo man after a stroke, presents with left sided ptosis and constricted pupil. He also has loss of pain and temp on the right side of his body and left side of his face. Which part of the brain is most likely affected?
a. Frontal cortex
b. Medulla
c. Cerebellum
d. Pons
e. Parietal cortex

A

The most likely affected part of the brain in this scenario is:

d. Pons

The symptoms described, including left-sided ptosis (drooping of the eyelid) and constricted pupil (miosis) on the same side, along with loss of pain and temperature sensation on the opposite side of the body and the same side of the face, are indicative of a classic presentation of a lateral medullary syndrome, also known as Wallenberg syndrome. This syndrome typically results from a lesion affecting the lateral portion of the medulla oblongata, which contains important structures involved in the pathway for sensation and autonomic control, including the descending sympathetic fibers responsible for maintaining normal pupil size.

How well did you know this?
1
Not at all
2
3
4
5
Perfectly
11
Q

A 47yo man has a temp of 39C and is delirious. He has developed blisters mainly on his trunk, which appeared a few hours ago. He is well and not on any medications. He last travelled 5 months ago to Italy. Which of the following is the most likely dx?
a. Shingles
b. Pemphigoid
c. Bullous pemphigus
d. Chicken pox

A

The most likely diagnosis in this scenario is:

d. Chicken pox

The combination of fever, delirium, and the development of blisters mainly on the trunk suggests a viral infection, with chickenpox (varicella-zoster virus) being the most likely cause. The patient’s recent travel to Italy is not relevant in this case, as chickenpox is a highly contagious viral infection that can occur worldwide. Additionally, the characteristic appearance of the rash with its distribution and timing aligns with chickenpox. It’s important to consider chickenpox, especially in adults who may not have been previously vaccinated or exposed to the virus. Prompt evaluation and appropriate management are essential to prevent complications and reduce transmission.

How well did you know this?
1
Not at all
2
3
4
5
Perfectly
12
Q

A 34yo alcoholic is found passed out in front of a local pub. The ambulance crew informs you that he was sweating when they found him and there were cans of cider lying empty around him. What is the initial stage of inv?
a. CT head
b. MRI head
c. Capillary blood sugar
d. ABG
e. MCV

A

The initial stage of investigation in this case would be:

c. Capillary blood sugar

Given the presentation of an alcoholic individual found passed out with sweating, the priority is to assess the blood sugar level to rule out hypoglycemia, which is common in individuals with alcohol use disorder. Hypoglycemia can result from alcohol-induced inhibition of gluconeogenesis and glycogenolysis, leading to decreased blood sugar levels. It’s essential to quickly assess and address any hypoglycemia to prevent complications and guide further management.

How well did you know this?
1
Not at all
2
3
4
5
Perfectly
13
Q

A young boy fell on his outstretched hand and has presented with pain around the elbow. He has absent radial pulse on the affected hand. What is the most likely dx?
a. Dislocated elbow
b. Undisplaced fx of radial head
c. Posterior dislocation of shoulder
d. Angulated supracondylar fx

A

The most likely diagnosis in this scenario is:

d. Angulated supracondylar fracture

The combination of pain around the elbow and an absent radial pulse suggests the possibility of an angulated supracondylar fracture. This type of fracture can cause compression or occlusion of the brachial artery, leading to compromised blood flow to the forearm and hand, resulting in an absent radial pulse. Prompt evaluation and management, including assessment of neurovascular status, are crucial to prevent complications such as compartment syndrome or ischemic injury to the hand.

How well did you know this?
1
Not at all
2
3
4
5
Perfectly
14
Q

A 32yo man presented with painless hematuria. He is hypertensive but the rest of the exam is unremarkable. What is the most likely dx?
a. TTP
b. Polycystic kidneys
c. Ca bladder
d. Ca prostate
e. HUS

A

The most likely diagnosis in this scenario is:

c. Bladder cancer

Painless hematuria in a hypertensive individual raises concern for underlying bladder cancer, especially considering the absence of significant findings on examination. While other conditions like polycystic kidneys and prostate cancer can also present with hematuria, bladder cancer is more commonly associated with painless hematuria, particularly in hypertensive patients. It’s important to further investigate with imaging studies and possibly cystoscopy to confirm the diagnosis and initiate appropriate management.

How well did you know this?
1
Not at all
2
3
4
5
Perfectly
15
Q

A 45yo female complains of pain in the inner side of her right thigh. She was dx with benign ovarian mass on the right. Which nerve is responsible for this pain?
a. Obturator nerve
b. Femoral nerve
c. Iliohypogastric nerve
d. Ovarian branch of splanchic nerve
e. Pudendal nerve

A

The nerve responsible for the pain in the inner side of the right thigh in this scenario is:

a. Obturator nerve

The obturator nerve innervates the inner thigh, and irritation or compression of this nerve can lead to pain in that region. The proximity of the ovarian mass to the obturator nerve can cause compression or irritation of the nerve fibers, resulting in referred pain to the inner thigh.

How well did you know this?
1
Not at all
2
3
4
5
Perfectly
16
Q

A 79yo stumbled and sustained a minor head injury 2 weeks ago. He has become increasingly confused, drowsy and unsteady. He has a GCS of 13. He takes warfarin for Afib. What is the most likely dx?
a. Subdural hemorrhage
b. Extradural hemorrhage
c. Subarachnoid hemorrhage
d. Cerebellar hemorrhage
e. Epidural hemorrhage

A

The most likely diagnosis in this case is:

a. Subdural hemorrhage

The patient’s history of a minor head injury, along with the onset of confusion, drowsiness, and unsteadiness, raises concern for a subdural hemorrhage. Subdural hemorrhages can occur following trauma, especially in older adults who may be on anticoagulant therapy like warfarin for conditions such as atrial fibrillation (Afib). The delayed onset of symptoms is characteristic of subdural hemorrhages, which can manifest days to weeks after the initial injury. Prompt evaluation and management, including neuroimaging, are crucial to assess the extent of the hemorrhage and initiate appropriate treatment.

How well did you know this?
1
Not at all
2
3
4
5
Perfectly
17
Q

A young college student is found in his dorm unconscious. He has tachyarrhythmia and high fever. He also seems to be bleeding from his nose, which on examination shows a perforated nasal septum. What is the most likely dx?
a. Marijuana OD
b. Heroin OD
c. Alcohol OD
d. CO poisoning
e. Cocaine OD

A

The most likely diagnosis in this case is:

e. Cocaine overdose

The combination of tachyarrhythmia, high fever, nasal septal perforation, and bleeding from the nose strongly suggests cocaine overdose. Cocaine can cause significant cardiovascular effects, including tachyarrhythmias such as ventricular tachycardia or fibrillation. Fever can result from the stimulant effects of cocaine, and nasal septal perforation is a characteristic finding associated with chronic cocaine use due to its vasoconstrictive properties leading to tissue necrosis. Prompt medical intervention, including supportive care and management of complications such as arrhythmias, is essential in cases of cocaine overdose.

How well did you know this?
1
Not at all
2
3
4
5
Perfectly
18
Q

In CRF, main cause of Vit D deficiency is the failure of:
a. Vit D absorption in intestines
b. 1 alpha hydroxylation of Vit D
c. 25 alpha hydroxylation of Vit D
d. Excess Vit D loss in urine
e. Availability of Vit D precursors

A

The main cause of vitamin D deficiency in chronic renal failure (CRF) is:

b. 1 alpha hydroxylation of vitamin D

In chronic renal failure, impaired renal function leads to reduced conversion of vitamin D into its active form, calcitriol, by the kidneys. This process occurs through the 1-alpha hydroxylation of vitamin D. Therefore, decreased synthesis of calcitriol contributes significantly to the development of vitamin D deficiency in individuals with chronic renal failure.

How well did you know this?
1
Not at all
2
3
4
5
Perfectly
19
Q

A woman who returned from abroad after 3 weeks of holiday complains of severe diarrhea of 3 weeks. She also developed IDA and folic acid def. What condition best describes her situation?

a. Malabsorption
b. Jejunal villous atrophy
c. Chronic diarrhea secretions
d. Increased catabolism
e. Increased secretions of acid

A

The condition that best describes the woman’s situation is:

a. Malabsorption

The combination of severe diarrhea lasting for three weeks, along with iron deficiency anemia (IDA) and folic acid deficiency, suggests malabsorption. Malabsorption syndromes can lead to inadequate absorption of nutrients, including iron and folic acid, resulting in deficiencies. The prolonged diarrhea further supports the possibility of malabsorption, as it can impair nutrient absorption in the intestines. Further evaluation would be needed to determine the specific cause of malabsorption, such as jejunal villous atrophy, but malabsorption is the overarching condition underlying her symptoms.

How well did you know this?
1
Not at all
2
3
4
5
Perfectly
20
Q

A man presented with cellulitis and swelling. He was started on flucloxacillin. What other medication do you want to add?
a. Vancomycin
b. Metronidazole
c. Ceftriaxone
d. Penicillin
e. Amoxicillin

A

In a case of cellulitis, especially if there’s a concern for methicillin-resistant Staphylococcus aureus (MRSA) infection, adding another antibiotic such as vancomycin would be prudent. Therefore, the additional medication to consider adding is:

a. Vancomycin

How well did you know this?
1
Not at all
2
3
4
5
Perfectly
21
Q

A 35yo man who has served in the army presents with lack of interest in enjoyable activities and feeling low. He doesn’t feel like reading the news or watching movies as he believes there is violence everywhere. What is the most appropriate first line therapy?
a. Citalopram
b. CBT
c. Lofepramine
d. Chlordiazepoxide
e. Desensitization

A

Given the symptoms described, particularly the lack of interest in enjoyable activities and the belief that there is violence everywhere, along with the patient’s history of military service, the most appropriate first-line therapy would likely involve addressing the underlying depressive symptoms with therapy rather than medication. Therefore, the most appropriate option is:

b. CBT (Cognitive Behavioral Therapy)

CBT is an evidence-based psychotherapy that focuses on identifying and challenging negative thought patterns and behaviors associated with depression. It can be particularly effective in addressing symptoms related to trauma and negative cognitions. Considering the patient’s concerns about violence and his history of military service, CBT could help him reframe his thoughts and cope with his symptoms more effectively.

How well did you know this?
1
Not at all
2
3
4
5
Perfectly
22
Q

A 12yo child with episodes of sudden bluish discoloration and brief loss of consciousness. Exam:
clubbing, central cyanosis, systolic thrill with systolic ejection murmur in 2nd left ICS. What is the most probable dx?
a. ASD
b. VSD
c. TOF
d. PDA
e. CoA

A

Based on the symptoms and examination findings described, the most probable diagnosis is:

c. TOF (Tetralogy of Fallot)

Tetralogy of Fallot is a congenital heart defect characterized by four abnormalities: pulmonary stenosis, overriding aorta, ventricular septal defect (VSD), and right ventricular hypertrophy. The episodes of sudden bluish discoloration (cyanotic spells or “tet” spells) and brief loss of consciousness (syncope) are classic presentations of TOF. The presence of clubbing, central cyanosis, and a systolic thrill with a systolic ejection murmur in the second left intercostal space further support this diagnosis.

How well did you know this?
1
Not at all
2
3
4
5
Perfectly
23
Q

Pt with hx of alcoholism, ataxic gait, hallucinations and loss of memory. He is given acamprosate. What other drug can you give with this?
a. Chlordiazepoxide
b. Diazepam
c. Disulfiram
d. Haloperidol
e. Thiamine

A

In a patient with a history of alcoholism experiencing symptoms such as ataxic gait, hallucinations, and loss of memory, the combination of acamprosate with another medication is often aimed at managing alcohol withdrawal symptoms and preventing relapse. Therefore, the most appropriate additional medication would be:

e. Thiamine

Thiamine supplementation is essential in patients with alcohol use disorder to prevent or treat Wernicke’s encephalopathy, a potentially serious neurological complication characterized by confusion, ataxia, and ophthalmoplegia. Thiamine deficiency is common in individuals with chronic alcoholism and can exacerbate neurological symptoms. Therefore, combining acamprosate with thiamine supplementation is a common and appropriate approach in the management of alcohol use disorder.

How well did you know this?
1
Not at all
2
3
4
5
Perfectly
24
Q

A 22yo woman with longstanding constipation has severe ano-rectal pain on defecation. Rectal exam: impossible due to pain and spasm. What is the most probable dx?
a. Anal hematoma
b. Anal abscess
c. Protalgia fugax
d. Anal fissure
e. Hemorrhoids

A

The most probable diagnosis in this case is:

d. Anal fissure

The symptoms of severe ano-rectal pain on defecation, along with the inability to perform a rectal exam due to pain and spasm, are highly suggestive of an anal fissure. Anal fissures are small tears or cracks in the lining of the anal canal, often caused by trauma during bowel movements, and they can result in intense pain, especially during defecation. The spasm of the anal sphincter muscles further exacerbates the pain and makes examination difficult. Anal fissures are a common cause of ano-rectal pain, particularly in individuals with longstanding constipation.

How well did you know this?
1
Not at all
2
3
4
5
Perfectly
25
Q

A 32yo had a normal vaginal delivery 10 days ago. Her uterus has involuted normally. Choose the single most likely predisposing factor for PPH?
a. Retained product
b. Uterine infection
c. DIC
d. Von Willebrand disease
e. Primary PPH

A

The single most likely predisposing factor for postpartum hemorrhage (PPH) in this case would be:

a. Retained product

Retained products of conception, such as placental fragments or membranes, can lead to postpartum hemorrhage by preventing the uterus from contracting adequately and achieving proper hemostasis. In this scenario, if the uterus has involuted normally but the patient still experiences postpartum hemorrhage, retained products of conception are a likely cause. Prompt identification and management of retained products are crucial to prevent complications associated with postpartum hemorrhage.

How well did you know this?
1
Not at all
2
3
4
5
Perfectly
26
Q

A butcher stabbed accidently his groin. He bled so much that the towel was soaked in blood and BP=80/50mmHg, pulse=130bpm. What % of circulatory blood did he lose?

a. <15%
b. 15-30%
c. 30-40%
d. 40-50%
e. >50%

A

Based on the given scenario of significant bleeding and the patient’s vital signs, the estimated blood loss can be categorized as follows:

BP=80/50 mmHg, pulse=130 bpm

These vital signs indicate hypotension and tachycardia, which are signs of compensatory mechanisms in response to significant blood loss.

Based on the clinical signs and symptoms, the estimated blood loss would likely fall within the range of:

e. >50%

The patient’s condition with significant hypotension and tachycardia suggests a massive hemorrhage, indicating a loss of more than 50% of circulatory blood volume. This level of blood loss is severe and requires urgent medical attention and resuscitation to prevent further complications and improve outcomes.

How well did you know this?
1
Not at all
2
3
4
5
Perfectly
27
Q

An old alcoholic presents with cough, fever, bilateral cavitating consolidation. What is the most probable cause?
a. AFB
b. Gram +ve diplococcic
c. Gram –ve cocci
d. Coagulase +ve cocci
e. Coagulase –ve cocci

A

The most probable cause of cough, fever, and bilateral cavitating consolidation in an old alcoholic is:

a. AFB (acid-fast bacilli)

This presentation is highly suggestive of tuberculosis, especially in an older alcoholic individual. Tuberculosis commonly presents with cough, fever, and pulmonary infiltrates, which can progress to cavitation, particularly in individuals with risk factors such as alcoholism. Acid-fast bacilli (AFB) are characteristic of Mycobacterium tuberculosis, the causative agent of tuberculosis, and are typically seen on sputum microscopy or culture in affected individuals. Therefore, AFB is the most probable cause in this case.

How well did you know this?
1
Not at all
2
3
4
5
Perfectly
28
Q

A 67yo man had successful thrombolysis for an inf MI 1 month ago and was discharged after 5days. He is now readmitted with pulmonary edema. What is the most probable dx?
a. Pulmonary stenosis
b. Aortic regurgitation
c. Ischemic mitral regurgitation
d. Mitral valve prolapse
e. Rheumatic mitral valve stenosis

A

The most probable diagnosis in this case is:

c. Ischemic mitral regurgitation

Ischemic mitral regurgitation occurs as a complication of myocardial infarction, particularly involving the papillary muscles or chordae tendineae. The dysfunction of these structures leads to incomplete closure of the mitral valve, resulting in regurgitation of blood from the left ventricle into the left atrium during systole. Pulmonary edema is a common manifestation of severe mitral regurgitation, as the backward flow of blood increases the volume and pressure within the left atrium and pulmonary vasculature, leading to pulmonary congestion and edema. Therefore, ischemic mitral regurgitation is the most probable diagnosis in this scenario, given the patient’s history of recent myocardial infarction and presentation with pulmonary edema.

How well did you know this?
1
Not at all
2
3
4
5
Perfectly
29
Q

A 3yo child who looks wasted on examination has a hx of diarrhea on and off. The mother describes the stool as bulky, frothy and difficult to flush. What is the single inv most likely to lead to dx?
a. Sweat chloride test
b. LFT
c. US abdomen
d. Anti-endomysial antibodies
e. TFT

A

The single investigation most likely to lead to the diagnosis in this case is:

a. Sweat chloride test

The presentation of a wasted appearance in a 3-year-old child with a history of chronic diarrhea, bulky, frothy stools, and difficulty flushing suggests the possibility of cystic fibrosis (CF). The sweat chloride test is the gold standard for diagnosing CF, as elevated chloride levels in sweat are characteristic of the condition. Therefore, performing a sweat chloride test would be the most appropriate initial investigation to confirm or rule out the diagnosis of CF in this child.

How well did you know this?
1
Not at all
2
3
4
5
Perfectly
30
Q

A girl with hx of allergies visited a friend’s farm. She got stridor, wheeze and erythematous rash. What is the most appropriate tx?
a. 0.25ml PO adrenaline
b. 0.25ml IM adrenaline
c. 0.25ml IV adrenaline
d. IV chlorphearamine

A

The most appropriate treatment for anaphylaxis, as described in this scenario, is:

b. 0.25ml IM adrenaline

Anaphylaxis is a severe allergic reaction that can lead to life-threatening symptoms such as stridor, wheezing, and erythematous rash. The first-line treatment for anaphylaxis is adrenaline (epinephrine) administered intramuscularly (IM). Adrenaline acts rapidly to reverse the symptoms of anaphylaxis by constricting blood vessels, relaxing bronchial smooth muscle, and improving cardiac output. Therefore, administering 0.25ml of IM adrenaline is the most appropriate immediate treatment in this case.

How well did you know this?
1
Not at all
2
3
4
5
Perfectly
31
Q

A 25yo had an LSCS 24h ago for fetal distress. She now complains of intermittent vaginal bleeding. Observations: O2 sat=98% in air, BP=124/82mmHg, pulse=84bpm, temp=37.8C. The midwife tells you that she had a retained placenta, which required manual removal in the OT. Choose the most appropriate C-Section complication in this case?
a. Uterine rupture
b. Retained POC
c. Aspiration pneumonitis
d. Endometritis
e. DIC

A

The most appropriate C-Section complication in this case, given the history of retained placenta requiring manual removal in the operating theater and the current presentation of intermittent vaginal bleeding, is:

e. DIC (Disseminated Intravascular Coagulation)

Disseminated intravascular coagulation (DIC) can occur as a complication of retained products of conception (POC), such as retained placenta. Manual removal of the placenta can lead to excessive bleeding and disruption of normal clotting mechanisms, resulting in DIC. The intermittent vaginal bleeding in this case may be indicative of ongoing bleeding secondary to DIC. Therefore, DIC is the most appropriate complication to consider in this scenario.

How well did you know this?
1
Not at all
2
3
4
5
Perfectly
32
Q

A 20yo man has a head on collision in a car. On presentation his is breathless, has chest pain and fx of 5-7th rib. CXR confirms this. What is the most appropriate initial action in this pt?

a. Antibiotics
b. Analgesia
c. O2 by mask
d. Physiotherapy
e. Refer to surgeon

A

The most appropriate initial action in this patient, who presents with breathlessness, chest pain, and confirmed rib fractures on CXR after a head-on collision, is:

c. O2 by mask

Given the potential for underlying lung injury or pneumothorax secondary to rib fractures and the patient’s symptoms of breathlessness, providing oxygen by mask is crucial to optimize oxygenation and prevent hypoxia. This action takes priority in the initial management of trauma patients with suspected chest injuries. After ensuring adequate oxygenation, further assessment and management, including analgesia and referral to a surgeon if necessary, can be initiated. However, oxygen therapy is the most immediate and appropriate initial action in this scenario.

How well did you know this?
1
Not at all
2
3
4
5
Perfectly
33
Q

A 30yo man presents with deep penetrating knife wound. He said he had TT when he left school.
What will you do for him now?
a. Human Ig and full course of tetanus vaccine
b. Antibiotic
c. Human Ig and TT
d. Human Ig only
e. Full course of tetanus vaccine only

A

The most appropriate action for the 30-year-old man with a deep penetrating knife wound who reports having received tetanus toxoid (TT) vaccination when he left school is:

b. Antibiotic

While tetanus prophylaxis is important in cases of penetrating wounds, if the patient has received a complete primary vaccination series and a booster within the past 10 years, additional tetanus toxoid vaccination may not be necessary. However, administering antibiotics is crucial to prevent wound infection and other potential complications. Therefore, providing antibiotics would be the most appropriate immediate action for this patient.

How well did you know this?
1
Not at all
2
3
4
5
Perfectly
34
Q

A mother comes with her 15m child. Which of the following will bother you?
a. Shies away from strangers
b. She can’t make a sentence
c. Can walk but not run
d. Vocabulary consists of only 2 meaningless words
e. None

A

The option that would likely be of concern is:

b. She can’t make a sentence

At 15 months of age, most children should be able to make basic sentences consisting of a few words. The inability to form sentences could be a developmental red flag and may indicate a language delay or other developmental issue. Therefore, this option would be the one that raises concern.

How well did you know this?
1
Not at all
2
3
4
5
Perfectly
35
Q

A 46yo man, known case of chronic GN presents to OPD. He feels well. BP = 140/90mmHg. Urine dipstick: protein ++, blood ++ and serum creatinine=106mmol/L. Which medication can prevent the progression of this dx?
a. Diuretics
b. ACEi
c. Cytotoxic meds
d. Longterm antibiotics
e. Steroids

A

The medication that can help prevent the progression of chronic glomerulonephritis (GN) in this case is:

b. ACEi (Angiotensin-Converting Enzyme inhibitor)

ACE inhibitors have been shown to be effective in slowing the progression of chronic kidney disease, including chronic glomerulonephritis. They help by reducing intraglomerular pressure and proteinuria, thereby protecting the kidneys from further damage and slowing the decline in kidney function. Therefore, prescribing an ACE inhibitor would be a suitable choice to help manage this patient’s condition and prevent progression.

How well did you know this?
1
Not at all
2
3
4
5
Perfectly
36
Q

A 78yo lady on warfarin for atrial fibrillation lives in a care home. She presents with hx of progressive confusion for three days. She was also noticed to have bruises on her arms. INR = 7. What is the most probable dx?
a. Alzheimers
b. Delirium
c. Vascular dementia
d. Subdural hemorrhage
e. Pick’s dementia

A

The most probable diagnosis in this case is:

d. Subdural hemorrhage

The presentation of progressive confusion, bruises on the arms, and an elevated INR level in a patient on warfarin suggests the possibility of a subdural hemorrhage. Warfarin predisposes patients to bleeding events, and an elevated INR increases the risk of hemorrhage, including subdural hemorrhage, which can present with symptoms such as confusion and bruising. Therefore, subdural hemorrhage is the most probable diagnosis in this scenario.

How well did you know this?
1
Not at all
2
3
4
5
Perfectly
37
Q

Mrs. A, a 32-year-old woman of African descent, presents to the clinic with complaints of recurrent episodes of severe pain in her joints and abdomen. She reports a long history of similar episodes since childhood. She mentions experiencing fatigue, shortness of breath, and occasional dizziness. Her medical history reveals frequent hospitalisations for pain management.

⚠️ Clinical Examination:

Pallor
Jaundice
Splenomegaly
Tenderness over long bones
Limited range of motion due to pain
📋 Investigations:

Haemoglobin: 70 g/L
Reticulocyte Count: 8%
Serum Bilirubin: 43.4 µmol/L
Blood Smear: Presence of Howell-Jolly bodies
❓ Question: What do you think this condition is?

Sickle Cell Anemia
Thalassemia
Hemophilia
Polycythemia Vera

A

The clinical presentation and investigation results are highly suggestive of:

Sickle Cell Anemia

Explanation:

•	Recurrent episodes of severe pain in the joints and abdomen: These pain crises are characteristic of sickle cell anemia due to vaso-occlusion.
•	Fatigue, shortness of breath, and occasional dizziness: These symptoms are consistent with chronic hemolytic anemia, common in sickle cell disease.
•	Pallor and jaundice: These signs indicate hemolysis.
•	Splenomegaly and tenderness over long bones: Splenomegaly is common in children with sickle cell anemia due to sequestration crises, though the spleen may become fibrotic and reduce in size over time. Bone pain and tenderness are frequent due to bone infarctions.
•	Limited range of motion due to pain: This is typical during pain crises.
•	Haemoglobin: 70 g/L: This is indicative of severe anemia.
•	Reticulocyte Count: 8%: Elevated reticulocyte count reflects increased red blood cell production in response to hemolysis.
•	Serum Bilirubin: 43.4 µmol/L: Elevated bilirubin is a result of increased breakdown of red blood cells.
•	Blood Smear: Presence of Howell-Jolly bodies: Howell-Jolly bodies indicate asplenia or hyposplenia, often seen in sickle cell anemia due to repeated splenic infarctions.

Therefore, the correct diagnosis is:
Sickle Cell Anemia

How well did you know this?
1
Not at all
2
3
4
5
Perfectly
38
Q

A 70-year-old woman presents with painless obstructive jaundice and a distended gallbladder. Ultrasound has shown a solid 3 cm mass arising from the head of the pancreas.
Which part of the extrahepatic biliary tree is being compressed by the mass?
A Common hepatic duct
B Infraduodenal common bile duct
C Left extrahepatic duct
D Right extrahepatic duct
E Supraduodenal common bile duct

A

The mass arising from the head of the pancreas is most likely compressing the supraduodenal common bile duct.

This is because the head of the pancreas lies closely adjacent to the common bile duct, specifically the supraduodenal part, which is the portion of the common bile duct that runs above the duodenum before it enters and runs behind the pancreas. Compression of this duct by a pancreatic head mass is a common cause of obstructive jaundice.

Therefore, the correct answer is:

E Supraduodenal common bile duct

How well did you know this?
1
Not at all
2
3
4
5
Perfectly
39
Q

A 45-year-old man presents with a lump protruding from his abdomen into his groin and the upper part of the scrotum.
Through which one of the following spaces would the lump have extruded out of the peritoneal cavity?
A Deep inguinal ring
B Femoral ring
C Inguinal triangle
D Interparietal space
E Superficial inguinal ring

A

The description of a lump protruding from the abdomen into the groin and upper part of the scrotum suggests an inguinal hernia. Specifically, for the hernia to reach the scrotum, it must pass through the inguinal canal. The entrance to the inguinal canal from the abdominal cavity is the deep inguinal ring.

Therefore, the correct answer is:

A Deep inguinal ring

How well did you know this?
1
Not at all
2
3
4
5
Perfectly
40
Q

A 55-year-old woman has been diagnosed with an advanced carcinoma of the stomach. She complains of a discharging swelling over her belly button.
Which one of the following intra abdominal ligaments is the portal of cause of this ulcer?
A Coronary ligament of the liver
B Left triangular ligament
C Lesser omentum
D Ligamentum teres
E Urachus

A

The discharging swelling over the belly button (umbilicus) in a patient with advanced carcinoma of the stomach suggests a Sister Mary Joseph nodule, which is a metastatic tumor deposit at the umbilicus. The pathway through which such metastases can occur is often related to the ligamentum teres, which is a remnant of the umbilical vein and runs from the umbilicus to the liver.

Therefore, the correct answer is:

D Ligamentum teres

How well did you know this?
1
Not at all
2
3
4
5
Perfectly
41
Q

A 25-year-old man, a cyclist, presents after being hit on the left side of his lower chest when involved in a collision with a car. He complains of pain in his left lower chest, left upper abdomen and left shoulder tip. He has full range of movements of his left upper limb. A FAST (focused abdominal sonography in trauma) scan shows free fluid under the left hemidiaphragm.
Which one of the following anatomical factors is the most likely cause of his left shoulder tip pain?
A Diaphragmatic rupture causing pain through the intercostobrachial nerve
B Left acromion fracture causing pain through the long thoracic nerve
C Left kidney rupture causing pain through the vagus nerve
D Left lower rib fractures causing pain from the lower six intercostal nerves
E Splenic rupture causing pain through the phrenic nerve

A

The patient’s presentation with pain in the left shoulder tip, left lower chest, and left upper abdomen, along with free fluid under the left hemidiaphragm on a FAST scan, strongly suggests splenic injury. The referred pain to the left shoulder tip is characteristic of Kehr’s sign, which occurs due to irritation of the diaphragm. The phrenic nerve, which innervates the diaphragm, originates from cervical nerves C3-C5 and can refer pain to the shoulder tip.

Therefore, the correct answer is:

E Splenic rupture causing pain through the phrenic nerve

How well did you know this?
1
Not at all
2
3
4
5
Perfectly
42
Q

A 30-year-old woman is to have postoperative analgesia by means of a nerve block following an operation for a fistula-in-ano.
Which one of the following nerves is the anaesthetist going to block by local anaesthetic?
A Common peroneal nerve
B Lumbosacral trunk
C Nerve to levator ani and external anal sphincter
D Obturator nerve
E Pudendal nerve

A

For postoperative analgesia following an operation for a fistula-in-ano, the most appropriate nerve to block is the pudendal nerve. The pudendal nerve provides sensory innervation to the perineum and the external genitalia, as well as motor innervation to the external anal sphincter and other pelvic floor muscles. Blocking this nerve will provide effective pain relief for procedures involving the anal and perineal region.

Therefore, the correct answer is:

E Pudendal nerve

The pudendal nerve (S2, S3 and S4) in the perineum is the one that can be blocked by local anaesthetic. The nerve is accessed through the lateral wall of the vagina
to produce anaesthesia to the perineal and anal skin. The nerve is formed by the anterior divisions of the ventral rami of the 2nd, 3rd and 4th sacral nerves. After its origin, it leaves the pelvis through the greater sciatic foramen to enter the gluteal region near the ischial spine. The nerve accompanies the internal pudendal vessels into the pudendal (Alcock’s) canal on the lateral wall of the ischiorectal fossa. In the posterior part of the canal it gives off its branches – inferior rectal nerve, the perineal nerve and the dorsal nerve of clitoris or penis.
The common peroneal nerve is formed by L4, L5 and S1 and S2. The lumbosacral trunk is constituted by L4 and L5, and joins the sacral plexus. S4 contributes to the nerve supplying the levator ani and external anal sphincter. The obturator nerve arises from the lumbar plexus from the anterior divisions of L2, L3 and L4. None of these nerves is the target of the caudal block.

How well did you know this?
1
Not at all
2
3
4
5
Perfectly
43
Q

A 42-year-old man, a labourer, while at work, suddenly develops severe pain in his lower back radiating to the buttocks, back of thigh, lower leg and sole of foot. He cannot feel when he sits as he has diminished sensation on his buttocks.
Which one of the following nerves is most likely affected?
A L5
B S1
C S2
D S3
E S4

A

The S3 nerve root is responsible for the sensation of the sitting area of buttock. It is a constituent of the sacral plexus. The most important nerve arising from the sacral plexus is the sciatic nerve. It arises from L4, L5, S1, S2 and S3 roots of the sacral plexus. At its origin it is 2 cm wide and is the thickest nerve in the body. It enters the gluteal region from the pelvis through the greater sciatic foramen. At a variable level in the back of the thigh proximal to the popliteal fossa it divides into the common peroneal (fibular, L4, L5, S1, S2 and tibial, L4, L5, S1, S2, S3) nerves. The surface anatomy is an imaginary line drawn from the midpoint of the ischial tuberosity and greater trochanter to the apex of the popliteal fossa formed by the junction of the semimembranosus and semitendinosus medially and biceps femoris laterally. As an aid to remember the dermatome levels, we stand mainly on S1 (sole of foot), sit on S3 (buttocks) and wipe S4 (immediate perianal area).
The commonest cause of damage to the sciatic nerve is iatrogenic misplaced
gluteal injection. It may be affected in pelvic disease, severe hip trauma (7% of dislocations and 16% of fracture dislocations), or after total hip replacement (1%). Complete sciatic nerve palsy is rare and results in a flail foot and severe difficulty in walking. Because of its anatomical location in close proximity to the fibular head, the common peroneal nerve is the commonest nerve to be damaged in the lower limb. This results in a foot drop, high stepping gait and sensory loss over the lower lateral part of the leg and dorsum of the foot.

How well did you know this?
1
Not at all
2
3
4
5
Perfectly
44
Q

A 65-year-old woman presents with severe pain on the inside of her thigh, which began 1 week ago. The pain radiates along the inside of the thigh to the knee. It is relieved by bending the hip and rotating it outwards. In that position a soft lump is palpable.
Which one of the following nerves is causing the pain?
A Femoral
B Genitofemoral
C Ilioinguinal
D Lateral femoral cutaneous
E Obturator

A

The symptoms described suggest that the obturator nerve is causing the pain. Here’s the reasoning:

•	Pain location and radiation: The obturator nerve supplies sensation to the inner thigh and knee area.
•	Pain relief by hip position: Bending the hip and rotating it outwards, which relieves the pain, indicates involvement of the obturator nerve.
•	Palpable lump: A palpable lump in this area could be associated with a hernia or other mass compressing the obturator nerve.

Answer:

E. Obturator

The obturator nerve is causing this patient’s pain. She has the clinical features of
an obturator hernia. The pain is referred to the knee by the geniculate branch of the obturator nerve (anterior divisions of L2, L3 and L4). The pain is much more pronounced in a strangulated hernia. Arising from the lumbar plexus, the obturator nerve lies on the psoas muscle and enters the obturator foramen. In the obturator canal it divides into anterior and posterior branches.
In an obturator hernia, a swelling is not often palpable unless the hip is abducted, flexed and externally rotated. The hernia can sometimes be felt as a tender swelling on rectal or vaginal examination.

Explanation:

The obturator nerve provides sensory innervation to the medial aspect of the thigh. Pain along the inner thigh to the knee, which is relieved by specific hip movements, is characteristic of obturator nerve involvement. The palpable lump further suggests a mass effect on this nerve.

How well did you know this?
1
Not at all
2
3
4
5
Perfectly
45
Q

A 30-year-old man presents with left ureteric colic. A spiral CT scan shows a stone impacted at the pelvic brim.
At which of the following anatomical sites would the stone be impacted?
A Common iliac artery bifurcation
B Fifth lumbar transverse process
C Ischial spine
D Pelviureteric junction
E Vas deferens crossing above the ureter

A

In the context of left ureteric colic with a stone impacted at the pelvic brim, the anatomical site where the stone is likely impacted is the common iliac artery bifurcation.

Answer:

A. Common iliac artery bifurcation

Explanation:

The ureter has several natural points of narrowing where stones are commonly impacted. One of these points is where the ureter crosses over the bifurcation of the common iliac artery at the pelvic brim. This anatomical feature makes it a common site for ureteral stones to become lodged, leading to ureteric colic symptoms.

The stone is impacted at the bifurcation of the common iliac artery where it leaves the psoas muscle. This is one of the points of natural narrowing where a stone may get arrested. The other points of natural narrowing are: pelviureteric junction, where it is crossed by the vas deferens or broad ligament and at the ureterovesical junction.
Knowledge of the relationships of the ureter is very important, so as to prevent iatrogenic damage. On the left it underlies the apex of the sigmoid mesocolon.
It then runs over the external iliac artery and vein and then down the side wall of the pelvis in front of the internal iliac artery and behind the ovary. On the right it will be in close proximity to a pelvic appendix. Further distally at the level of the ischial spine it travels forwards and medially to enter the bladder base. Here the vas deferens in the male crosses above the ureter and in the female it crosses the lateral vaginal fornix.

How well did you know this?
1
Not at all
2
3
4
5
Perfectly
46
Q

A 35-year-old woman is undergoing a hysterectomy with preservation of ovaries.
During the operation which one of the following anatomical structures is vulnerable to iatrogenic damage?
A Fallopian tubes
B Ovaries
C Rectum
D Ureters
E Urinary bladder

A

During a hysterectomy with preservation of the ovaries, the ureters are particularly vulnerable to iatrogenic damage.

Answer:

D. Ureters

Explanation:

The ureters are at risk during a hysterectomy because they run close to the uterine arteries, which need to be ligated during the procedure. The ureters pass underneath the uterine arteries (“water under the bridge”) and are at risk of being inadvertently cut, clamped, or ligated. This is a well-known complication of gynecological surgeries, including hysterectomy.

How well did you know this?
1
Not at all
2
3
4
5
Perfectly
47
Q

A 25-year-old man presents following a straddle injury to his perineum having fallen astride on the beam in the gymnasium. Clinically there is a perineal haematoma with blood on his external urinary meatus.
Which anatomical structure is most likely to be injured?
A Bladder neck
B Bulbar urethra
C Membranous urethra
D Prostatic urethra
E Urinary bladder

A

Given the presentation of a perineal hematoma and blood at the external urinary meatus following a straddle injury, the anatomical structure most likely to be injured is the bulbar urethra.

Answer:

B. Bulbar urethra

Explanation:

A straddle injury often results in trauma to the bulbar (or bulbous) part of the urethra, which is located in the perineum. This part of the urethra is vulnerable to direct trauma when a person falls astride an object. The presence of blood at the external urinary meatus is a typical sign of urethral injury, and in this context, the bulbar urethra is the most commonly affected segment.

This patient has injured his bulbar urethra (sometimes referred to as the anterior urethra) in the perineum. This injury involves the junction of the membranous with the bulbar portion of the urethra. The anatomy of this region is such that extravasation of urine occurs, unless recognition of the injury and treatment
is carried out promptly. Urine leaks between the perineal membrane and the membranous layer of the perineal fascia (Colles’ fascia). As both these layers are firmly attached to the ischiopubic rami posteriorly, urine extravasates anteriorly into the loose connective tissue around the scrotum, penis and anterior abdominal wall. Should the posterior urethra be injured, urine leaks into the pelvic extraperitoneal tissues. Tear of the perineal membrane results in extravasation in the perineum.
Anatomically, the membranous urethra is the shortest (1.5 cm) and least dilatable part of the male urethra (which is 18–20 cm long). The anterior urethra (16 cm) has a proximal perineal and a distal penile component. The posterior urethra is subdivided into preprostatic, prostatic and membranous parts. The female urethra is 4 cm long and 6 mm wide.
Injury to the bladder neck, membranous and prostatic urethra can occur in fractures of the pelvis. Bladder rupture, extra- or intraperitoneal, is highly unlikely with such an injury. The clinical findings are not those of a bladder rupture.

How well did you know this?
1
Not at all
2
3
4
5
Perfectly
48
Q

A 16-year-old boy presents with a condition, where he passes urine from a meatus on the underside of his glans penis, a situation he finds embarrassing.
Which one of the following congenital anatomical abnormality does he suffer from?
A Coronal hypospadias
B Epispadias
C Glandular hypospadias
D Penile hypospadias
E Perineal hypospadias

A

The condition described, where the boy passes urine from a meatus on the underside of his glans penis, is indicative of glandular hypospadias.

Answer:

C. Glandular hypospadias

Explanation:

Hypospadias is a congenital condition where the urethral opening is located on the underside of the penis rather than at the tip. Glandular hypospadias specifically refers to the urethral opening being located on the underside of the glans penis, which matches the description provided. This is one of the more common and less severe forms of hypospadias.

49
Q

A 35-year-old man is recovering from a severe motorcycle injury where he sustained major pelvic fractures and bladder and urethral injuries. He did not suffer any
head injuries. Six months after his accident his urethral catheter is removed. He is completely incontinent.
Which one of the following anatomical structures has been irreparably damaged, causing his incontinence?
A External urethral sphincter
B Internal urethral sphincter
C Membranous urethra
D Puboprostatic ligaments
E Urinary bladder rupture

A

Given the patient’s history of major pelvic fractures and bladder and urethral injuries, and his resulting complete incontinence after removal of the urethral catheter, it is most likely that the external urethral sphincter has been irreparably damaged.

Answer:

A. External urethral sphincter

Explanation:

The external urethral sphincter (also known as the sphincter urethrae) is a critical structure for voluntary control of urination. Damage to this sphincter, which can occur due to severe trauma or injury to the pelvic region, can result in loss of voluntary control over urination, leading to complete incontinence. The internal urethral sphincter contributes to continence but is primarily responsible for involuntary control, whereas the external sphincter is key for voluntary control, which aligns with the patient’s presentation of incontinence following catheter removal.

This person’s urinary incontinence is due to damage to the external urethral sphincter. The internal urethral sphincter surrounding the proximal prostatic urethra is not responsible for urinary continence. This helps to prevent retrograde ejaculation by closing off the bladder neck during ejaculation.

The urethral sphincter mechanism extends from the perineum through the urogenital hiatus into the pelvic cavity. The mechanism consists of the striated and smooth muscle of the urethra and the pubourethral part of levator ani; this surrounds the membranous urethra in the male. The fibres also reach up to the lowest part of the bladder neck. The muscles are of the slow twitch variety. The innervation is from the perineal branch of the pudendal nerve and the pelvic splanchnic nerves. All these nerves originate in the S2, S3 and S4 spinal segments.
The urethral sphincter mechanism compresses the urethra when the bladder contains urine. It is located around the region of the highest urethral closing pressure, thus playing an important role in maintaining urinary continence. It relaxes during micturition and contracts to expel final drops of urine or semen from the bulbar urethra.

50
Q

A 62-year-old man presents with shock, secondary to acute pancreatitis. A decision is made to place a femoral line as part of his initial resuscitation in the emergency department.
Which one of the following lies outwith the femoral sheath?
A Cloquet’s node
B Femoral artery
C Femoral canal
D Femoral nerve
E Femoral vein

A

The structure that lies outside (outwith) the femoral sheath is the femoral nerve.

Answer:

D. Femoral nerve

Explanation:

The femoral sheath is a fascial compartment located in the upper part of the thigh, containing the femoral artery, femoral vein, and the femoral canal, which includes Cloquet’s (or Rosenmüller’s) node. The femoral nerve, however, lies outside the femoral sheath, laterally to these structures. Therefore, it is not enclosed within the femoral sheath.

51
Q

A 75-year-old woman attends 6 months following left total hip arthroplasty through a lateral approach. She is found to have a left sided Trendelenburg gait with weakness of the ipsilateral abductors.
Which one of the following nerves is most likely to be injured, and explains the examination findings?
A Femoral nerve
B Inferior gluteal nerve
C Obturator nerve
D Sciatic nerve
E Superior gluteal nerve

A

The presence of a left-sided Trendelenburg gait with weakness of the ipsilateral hip abductors following a lateral approach total hip arthroplasty indicates injury to the superior gluteal nerve.

Answer:

E. Superior gluteal nerve

Explanation:

The superior gluteal nerve innervates the gluteus medius and gluteus minimus muscles, which are responsible for hip abduction. Damage to this nerve can result in weakness of these muscles, leading to a Trendelenburg gait, where the pelvis drops on the contralateral side when standing on the affected leg. This nerve is particularly susceptible to injury during surgical approaches to the hip, such as a lateral approach used in total hip arthroplasty.

52
Q

A 45-year-old man presents with a chronic swelling in the popliteal fossa consistent with a Baker’s cyst.
Of the following muscles that comprise the borders of the popliteal fossa, which one has an insertion on the head of the fibula?
A Biceps femoris
B Gastrocnemius
C Plantaris
D Semimembranosus
E Semitendinosus

A

The muscle that comprises the borders of the popliteal fossa and has an insertion on the head of the fibula is the biceps femoris.

Answer:

A. Biceps femoris

Explanation:

The biceps femoris muscle, which forms the superolateral boundary of the popliteal fossa, inserts on the head of the fibula. The other muscles mentioned do not insert on the head of the fibula:

•	Gastrocnemius: Inserts into the calcaneus via the Achilles tendon.
•	Plantaris: Also inserts into the calcaneus.
•	Semimembranosus: Inserts on the medial condyle of the tibia.
•	Semitendinosus: Inserts on the proximal tibia (pes anserinus).
53
Q

A 25-year-old man presents with significant pain and swelling to the anterior aspect of the leg following a direct blow during a football match. Clinical examination and compartment pressure monitoring confirm the diagnosis of acute compartment syndrome of the leg.

Which of the following structures is found within the lateral compartment of the leg?
A Anterior tibial artery
B Deep peroneal nerve
C Peroneus tertius
D Superficial peroneal nerve
E Tibialis anterior

A

The structure found within the lateral compartment of the leg is the superficial peroneal nerve.

Answer:

D. Superficial peroneal nerve

Explanation:

The lateral compartment of the leg contains the peroneus longus and peroneus brevis muscles, as well as the superficial peroneal nerve. The other structures listed are found in different compartments:

•	Anterior tibial artery: Found in the anterior compartment.
•	Deep peroneal nerve: Found in the anterior compartment.
•	Peroneus tertius: Found in the anterior compartment.
•	Tibialis anterior: Found in the anterior compartment.

The superficial peroneal nerve supplies the muscles in the lateral compartment and provides sensation to the dorsum of the foot and the lower part of the leg.

54
Q

A 68-year-old woman presents with a bimalleolar fracture of the right ankle. Operative fixation is undertaken using a direct lateral approach to the lateral malleolus, with a direct incision used for the medial malleolus.
Which one of the following structures is found anterior to the medial malleolus?
A Flexor digitorum longus tendon
B Posterior tibial artery
C Saphenous nerve
D Tibial nerve
E Tibialis posterior tendon

A

The structure found anterior to the medial malleolus is the saphenous nerve.

Answer:

C. Saphenous nerve

Explanation:

The saphenous nerve is a branch of the femoral nerve and provides sensory innervation to the medial side of the leg and foot. It runs anterior to the medial malleolus, making it vulnerable during surgical procedures involving this area. The other structures listed are located posterior to the medial malleolus:

•	Flexor digitorum longus tendon
•	Posterior tibial artery
•	Tibial nerve
•	Tibialis posterior tendon

These structures pass behind the medial malleolus in the tarsal tunnel.

55
Q

A 43-year-old woman presents with lumbar back pain and sciatica, with associated numbness over the lateral border of the right foot and ankle, along with an absent ipsilateral ankle jerk.
Which spinal nerve distribution does this represent?
A L4
B L5
C S1
D S2
E S3

A

The presentation of lumbar back pain, sciatica, numbness over the lateral border of the foot and ankle, and an absent ipsilateral ankle jerk is consistent with involvement of the S1 spinal nerve.

Answer:

C. S1

Explanation:

The S1 spinal nerve provides sensory innervation to the lateral aspect of the foot and ankle. It also contributes to the Achilles (ankle) reflex. Compression or irritation of the S1 nerve root can lead to the symptoms described, including sciatica, numbness in the specified area, and a diminished or absent ankle jerk.

56
Q

A 60-year-old man has been admitted to hospital unconscious after a car crash with multiple injuries and excessive bleeding in relation to his injuries. He is found to have atrial fibrillation and another occupant of the car (without serious injuries) reports that the unconscious man was taking treatment to prevent complications of his irregular heartbeat.
Which of the following is most likely to have caused the excessive bleeding?
A Deficient heparin level in the blood
B Deficient prothrombin level in the blood
C Excessive calcium level in the blood
D Excessive haematocrit
E Excessive vitamin D level in the blood

A

Given the patient’s history of atrial fibrillation and treatment to prevent complications of irregular heartbeat, the most likely cause of the excessive bleeding is a deficient prothrombin level in the blood, which can be due to the use of anticoagulant medications.

Answer:

B. Deficient prothrombin level in the blood

Explanation:

Patients with atrial fibrillation are commonly prescribed anticoagulants such as warfarin or direct oral anticoagulants (DOACs) to prevent thromboembolic complications like stroke. These medications work by inhibiting various components of the clotting cascade, leading to a reduction in prothrombin and other clotting factors. This inhibition can result in excessive bleeding, especially in the context of trauma.

•	Heparin deficiency (A) would not cause bleeding; in fact, heparin is an anticoagulant, and a deficiency would lead to increased clotting.
•	Excessive calcium levels (C) and excessive vitamin D levels (E) do not directly relate to coagulation in a way that would cause excessive bleeding in this context.
•	Excessive haematocrit (D) typically relates to thickened blood, not increased bleeding.

Therefore, the most plausible cause in this scenario is a deficient prothrombin level due to anticoagulant therapy.

57
Q

A 60-year-old woman is having open cholecystectomy which has been prolonged by technical diflculties. It is noted that her core temperature has fallen by over a degree since the start of surgery.
Which of the following is most likely to have caused the fall in core temperature?
A Reaction to propofol administered by the anaesthetist
B An operating theatre temperature in the range 25–30°C
C Intravenous fluids administered at around 35°C
D Loss of homeostasis due to the anaesthetic
E Patients’ increased metabolic rate

A

The most likely cause of the fall in core temperature during a prolonged open cholecystectomy is loss of homeostasis due to the anaesthetic.

Answer:

D. Loss of homeostasis due to the anaesthetic

Explanation:

Anesthesia disrupts the body’s normal thermoregulatory mechanisms, leading to a loss of homeostasis. This disruption can cause a significant drop in core temperature, known as perioperative hypothermia. Contributing factors include:

•	Vasodilation: Anesthetics cause vasodilation, which increases heat loss from the body surface.
•	Impaired thermoregulation: Anesthetics impair the body’s ability to regulate temperature effectively.
•	Exposure to a cool environment: The operating room is often kept cooler to maintain a sterile environment and for the comfort of the surgical team, even if the range provided (25–30°C) is not typically that cool.
•	Cool intravenous fluids and exposed internal organs: Even though fluids are administered at around 35°C, they can still contribute to heat loss, especially when internal organs are exposed during surgery.

The other options are less likely causes of significant hypothermia in this context:

•	Reaction to propofol (A) typically does not cause a significant drop in core temperature.
•	Operating theatre temperature (B) in the range of 25–30°C would not normally cause a significant fall in core temperature unless combined with other factors.
•	Intravenous fluids at around 35°C (C) are usually warmed to help maintain body temperature.
•	Patients’ increased metabolic rate (E) would generate more heat, not less.

Therefore, the most likely cause of the observed drop in core temperature is the loss of thermoregulatory homeostasis due to the effects of anesthesia.

58
Q

A 60-year-old man has been admitted for repair of an inguinal hernia. He has been on insulin therapy for many years and now su ffers from orthostatic hypotension due to diabetic autonomic neuropathy. His arterial blood pressure when lying down is 130/80 mmHg, but when he stands for five minutes it falls to 105/70 mmHg and he feels somewhat light-headed.
Which of the following is most likely to have caused the fall in his blood pressure on standing?
A Excessive blood pressure homeostasis
B Excessive variability of his resting heart rate due to sinus arrhythmia
C Excessive venoconstriction
D Impaired sympathetically-induced peripheral vasoconstriction
E Impairment of his renin-angiotensin system

A

The most likely cause of the fall in blood pressure on standing in this patient with diabetic autonomic neuropathy is impaired sympathetically-induced peripheral vasoconstriction.

Answer:

D. Impaired sympathetically-induced peripheral vasoconstriction

Explanation:

Orthostatic hypotension in diabetic patients is often due to autonomic neuropathy, which affects the body’s ability to regulate blood pressure upon standing. In a healthy individual, standing up triggers sympathetic nervous system activity that causes vasoconstriction, which helps maintain blood pressure. In patients with diabetic autonomic neuropathy, this sympathetic response is impaired, leading to insufficient vasoconstriction and a subsequent drop in blood pressure upon standing.

The other options are less likely causes:

•	Excessive blood pressure homeostasis (A) would actually help maintain blood pressure, not cause a drop.
•	Excessive variability of his resting heart rate due to sinus arrhythmia (B) is unrelated to the drop in blood pressure on standing.
•	Excessive venoconstriction (C) would help maintain or even increase blood pressure, not cause it to drop.
•	Impairment of his renin-angiotensin system (E) could contribute to chronic hypotension but is less directly involved in the acute orthostatic response compared to sympathetic vasoconstriction.

Therefore, the most plausible cause of the orthostatic hypotension in this context is the impaired sympathetically-induced peripheral vasoconstriction due to diabetic autonomic neuropathy.

59
Q

A 55-year-old woman with multiple injuries has developed disseminated intravascular coagulation leading to hypofibrinogenaemia and a risk of excessive haemorrhage.
Which of the following is most likely to have caused this condition?
A Activation of the extrinsic clotting pathway by a drug (extrinsic agent) administered to her
B Activation of the extrinsic clotting pathway by products of tissue damage
C Activation of the intrinsic clotting pathway by excess factor VIII levels
D Activation of the intrinsic clotting pathway by excess calcium ions
E Excess production of prothrombin (factor II)

A

The most likely cause of disseminated intravascular coagulation (DIC) leading to hypofibrinogenemia and a risk of excessive hemorrhage in a patient with multiple injuries is activation of the extrinsic clotting pathway by products of tissue damage.

Answer:

B. Activation of the extrinsic clotting pathway by products of tissue damage

Explanation:

Disseminated intravascular coagulation (DIC) is a complex condition characterized by widespread activation of the coagulation cascade, leading to the formation of blood clots throughout the body’s small vessels. This process consumes clotting factors and platelets, leading to a paradoxical risk of both thrombosis and bleeding.

In the context of multiple injuries, the extrinsic clotting pathway is activated by tissue factor (TF), which is released from damaged tissues. This pathway is a primary mechanism by which DIC is initiated in trauma and other conditions involving significant tissue injury.

•	Activation of the extrinsic clotting pathway by a drug (A) is less likely as the cause in the context of trauma unless a specific drug known to induce DIC is involved.
•	Activation of the intrinsic clotting pathway by excess factor VIII levels (C) and by excess calcium ions (D) are not typical mechanisms leading to DIC in trauma patients.
•	Excess production of prothrombin (factor II) (E) does not typically cause DIC; rather, DIC involves the consumption of clotting factors, including prothrombin.

Therefore, the most plausible cause of DIC in a trauma patient is the activation of the extrinsic clotting pathway by products of tissue damage.

60
Q

A 25-year-old woman presents with gangrenous changes in both legs. She reports that she has been living rough and has suffered greatly from the recent extremely cold spell (around –10°C at night).
Which of the following is most likely to have caused this condition?
A Abnormal temperature-regulating centre in the basal ganglia
B Cerebrally-induced hypothermia
C Core temperature maintained by peripheral vasoconstriction
D Disorder of the autonomic nerves
E Leg skin temperature has been only 20–25°C

A

Given the patient’s history of exposure to extremely cold temperatures and the resulting gangrenous changes in her legs, the most likely cause of this condition is core temperature maintained by peripheral vasoconstriction.

Answer:

C. Core temperature maintained by peripheral vasoconstriction

Explanation:

When the body is exposed to cold environments, it attempts to maintain core temperature by constricting blood vessels in the periphery (including the legs) to reduce heat loss. Prolonged and severe peripheral vasoconstriction in response to cold exposure can lead to reduced blood flow to the extremities, resulting in tissue ischemia and potentially gangrene.

•	Abnormal temperature-regulating centre in the basal ganglia (A) is unlikely to be the cause in this context, as the basal ganglia are not primarily involved in thermoregulation.
•	Cerebrally-induced hypothermia (B) is not a typical mechanism leading to gangrene in cold exposure scenarios.
•	Disorder of the autonomic nerves (D) could theoretically affect peripheral circulation, but the primary issue here is the body’s natural response to cold.
•	Leg skin temperature has been only 20–25°C (E) indicates a low temperature but does not directly explain the mechanism leading to gangrene.

Therefore, the most plausible cause of the gangrenous changes in this patient’s legs is the peripheral vasoconstriction that occurred as her body attempted to maintain core temperature in extremely cold conditions.

61
Q

Two 40-year-old men have just been admitted to the emergency department, one with immersion hypothermia and the other with haematemesis. Both have relatively cool peripheries and contracted superficial veins, from which it is difficult to obtain a blood sample.
Which of the following is most likely to differ between the two men?
A The level of activity in peripheral sympathetic nerves
B The part of the brain initiating vasoconstriction
C Heat exchange between forearm arteries and venae commitantes
D Peripheral catecholamine receptors
E Total forearm and hand blood flow

A

In this scenario, the key difference between the two men—one with immersion hypothermia and the other with haematemesis—is likely the total forearm and hand blood flow.

Answer:

E. Total forearm and hand blood flow

Explanation:

Both men have cool peripheries and contracted superficial veins, which can be a response to peripheral vasoconstriction. However, the underlying causes of vasoconstriction are different:

•	Immersion hypothermia leads to vasoconstriction primarily to conserve core body heat. This response decreases blood flow to the periphery to minimize heat loss.
•	Haematemesis (vomiting blood) can lead to vasoconstriction due to blood loss and subsequent hypovolemic shock. This response aims to maintain blood pressure and ensure adequate perfusion to vital organs.

While both conditions result in peripheral vasoconstriction, the total forearm and hand blood flow would likely differ due to the distinct underlying mechanisms. The hypothermic patient will have reduced blood flow due to the body’s attempt to conserve heat, whereas the patient with haematemesis may have reduced blood flow due to compensatory mechanisms in response to blood loss and hypovolemia.

•	The level of activity in peripheral sympathetic nerves (A) might be similar in both cases as vasoconstriction is generally mediated by sympathetic activity.
•	The part of the brain initiating vasoconstriction (B) is typically the hypothalamus for both thermal regulation and volume control.
•	Heat exchange between forearm arteries and venae commitantes (C) would not be significantly different as this is more of a passive process.
•	Peripheral catecholamine receptors (D) would likely respond similarly in both situations to sympathetic stimulation.

Therefore, the most likely difference between the two men lies in the total forearm and hand blood flow, influenced by the different physiological stressors they are experiencing.

62
Q

An 18-year-old woman presents with severely excessive sweating in her hands, which interferes with her daily activities.
Which of the following is most likely to have caused this condition?
A Emotional instability
B Over-activity of cholinergic parasympathetic nerves
C Over-activity of cholinergic sympathetic nerves
D Over-activity of temperature sensitive receptors in her hands
E Under-activity of noradrenergic sympathetic nerves

A

The condition described, characterized by severely excessive sweating in the hands, is most likely due to over-activity of cholinergic sympathetic nerves.

Answer:

C. Over-activity of cholinergic sympathetic nerves

Explanation:

Hyperhidrosis, particularly in the hands (palmar hyperhidrosis), is typically caused by an over-activity of the sympathetic nervous system. The sweat glands in the skin are primarily innervated by sympathetic nerves that release acetylcholine, a cholinergic neurotransmitter. This condition is not related to parasympathetic nerves, temperature-sensitive receptors in the hands, nor a deficiency in noradrenergic sympathetic activity.

•	Emotional instability (A) can exacerbate sweating but is not the primary cause of hyperhidrosis.
•	Over-activity of cholinergic parasympathetic nerves (B) is incorrect because sweat glands are innervated by the sympathetic nervous system.
•	Over-activity of temperature-sensitive receptors in her hands (D) does not directly cause hyperhidrosis; these receptors regulate responses to temperature, not sweating.
•	Under-activity of noradrenergic sympathetic nerves (E) would not cause excessive sweating; rather, it would reduce adrenergic responses.

Thus, the most likely cause of her condition is the over-activity of cholinergic sympathetic nerves.

63
Q

A 30-year old woman is attending the gastroenterology clinic with a complaint of severe diarrhoea and weight loss. While waiting in the clinic, she is sweating profusely. Her temperature is normal and pulse rate 110 beats per minute. The temperature in the clinic room is 22°C.

Which of the following is the most likely diagnosis?
A Anxiety neurosis
B Hyperhidrosis
C Inflammatory bowel disease
D Primary thyrotoxicosis
E Sepsis

A

Given the symptoms of severe diarrhea, weight loss, profuse sweating, normal temperature, and an elevated pulse rate, the most likely diagnosis is primary thyrotoxicosis.

Answer:

D. Primary thyrotoxicosis

Explanation:

Primary thyrotoxicosis (hyperthyroidism) is a condition where there is excessive production of thyroid hormones. This can lead to symptoms such as weight loss, increased bowel movements (which can present as severe diarrhea), and increased sweating (hyperhidrosis). The elevated pulse rate (tachycardia) is also a common feature of hyperthyroidism.

•	Anxiety neurosis (A) could cause sweating and an elevated pulse, but it is less likely to explain the severe diarrhea and significant weight loss.
•	Hyperhidrosis (B) causes excessive sweating but does not account for the other symptoms like severe diarrhea and weight loss.
•	Inflammatory bowel disease (C) can cause severe diarrhea and weight loss but is less likely to cause profuse sweating and a rapid pulse without a fever.
•	Sepsis (E) could cause sweating and an elevated pulse, but it would typically also cause fever and other signs of systemic infection.

Therefore, primary thyrotoxicosis is the most consistent diagnosis given the combination of symptoms.

64
Q

A 25-year-old woman suffers intermittently from cold white hands, particularly in cold weather. During a test period of body warming in a cool environment her hand skin temperature rises, after some time, from 20°C to 35°C on both sides, her pulse rate rises from 65 to 90 beats per minute, and her blood pressure from 115/80 mmHg to 135/80 mmHg.
What is the most likely diagnosis?
A Bilateral arterial disease in the arms
B Early heart failure
C Excessive and dangerous body warming
D Intermittent vasospasm related to sympathetic nervous activity
E Thyrotoxicosis

A

The most likely diagnosis for the woman who suffers intermittently from cold white hands, particularly in cold weather, is intermittent vasospasm related to sympathetic nervous activity.

Answer:

D. Intermittent vasospasm related to sympathetic nervous activity

Explanation:

The symptoms described, including intermittent cold and white hands that improve with warming, are characteristic of Raynaud’s phenomenon. This condition involves episodic vasospasm of the arteries and arterioles in the extremities, usually triggered by cold or stress, leading to reduced blood flow. When the vasospasm resolves, the hands warm up and the skin temperature increases. The rise in pulse rate and blood pressure during the warming period suggests a systemic response to the cold and subsequent warming.

•	Bilateral arterial disease in the arms (A) would likely cause persistent symptoms rather than intermittent ones and might not improve significantly with warming.
•	Early heart failure (B) would typically present with other symptoms such as shortness of breath, fatigue, and fluid retention, rather than isolated intermittent cold hands.
•	Excessive and dangerous body warming (C) would generally cause symptoms of overheating, such as sweating, heat exhaustion, or even heat stroke, rather than an improvement in hand temperature.
•	Thyrotoxicosis (E) typically presents with symptoms like weight loss, heat intolerance, and palpitations, rather than intermittent cold hands.

Therefore, the intermittent vasospasm related to sympathetic nervous activity (Raynaud’s phenomenon) is the most consistent diagnosis given the patient’s symptoms and response to warming.

65
Q

A 16-year-old man is one of a group of young people who have been rescued from a hill walk after they were caught in mist, rain and wind, the environmental temperature being about 5°C. He has cold peripheries and looks exhausted, but can answer questions rationally.
Which of the following is most likely to have caused his condition?
A Early stage of infection B Frostbite
C Hypoglycaemia
D Hypothermia
E Ventricular fibrillation

A

Given the context of being exposed to mist, rain, wind, and low temperatures (about 5°C), the most likely cause of the young man’s condition (cold peripheries, exhaustion, but still rational) is hypothermia.

Answer:

D. Hypothermia

Explanation:

Hypothermia occurs when the body loses heat faster than it can produce it, leading to a drop in core body temperature. Symptoms of mild hypothermia include shivering, cold peripheries, exhaustion, and impaired physical and mental function, but the person can still be rational and answer questions.

•	Early stage of infection (A) might cause exhaustion and possibly fever or chills, but the environmental context makes hypothermia more likely.
•	Frostbite (B) affects localized areas exposed to cold and results in numbness and discoloration, but does not explain the generalized exhaustion and cold peripheries.
•	Hypoglycaemia (C) could cause confusion and cold sweat, but the primary context here involves prolonged exposure to cold weather, making hypothermia the more likely cause.
•	Ventricular fibrillation (E) is a life-threatening heart rhythm problem that would cause unconsciousness or severe distress, not rational responses.

Therefore, hypothermia is the most plausible cause given the circumstances of exposure to cold weather and the symptoms described.

66
Q

A 24-year-old man has fallen two storeys from the roof of a house. He has a Glasgow coma scale 15 and is complaining of pain in his back. On arrival in the emergency department, he is hypotensive with warm peripheries despite 2 L of Hartmann’s solution. Following primary and secondary survey, the only injury identified on a CT scan is a burst fracture of T10 with significant impingement of the spinal canal.
In neurogenic shock, which is the most important contributor to hypotension?
A Decreased parasympathetic tone
B Decreased circulating volume
C Increased sympathetic tone
D Increased systemic vascular resistance
E Increased venous capacitance

A

In the context of neurogenic shock due to a burst fracture of T10 with significant impingement of the spinal canal, the most important contributor to hypotension is increased venous capacitance.

Answer:

E. Increased venous capacitance

Explanation:

Neurogenic shock results from the disruption of the autonomic pathways in the spinal cord, leading to a loss of sympathetic tone. This causes widespread vasodilation and increased venous capacitance (i.e., pooling of blood in the venous system), resulting in hypotension despite adequate fluid resuscitation. Key factors in neurogenic shock include:

•	Decreased parasympathetic tone (A): Neurogenic shock is primarily due to the loss of sympathetic tone, not parasympathetic tone.
•	Decreased circulating volume (B): While hypovolemia can cause hypotension, the patient is described as having warm peripheries, which suggests vasodilation rather than hypovolemic shock.
•	Increased sympathetic tone (C): Neurogenic shock is characterized by decreased sympathetic tone, not increased.
•	Increased systemic vascular resistance (D): In neurogenic shock, systemic vascular resistance is decreased due to vasodilation.
•	Increased venous capacitance (E): The loss of sympathetic tone leads to venous pooling and increased capacitance, which reduces venous return to the heart and lowers cardiac output, resulting in hypotension.

Therefore, the increased venous capacitance is the most critical factor contributing to the hypotension seen in neurogenic shock.

67
Q

An 84-year-old man is in the surgical high dependency unit the morning following laparotomy for a perforated duodenal ulcer. He had an episode of chest pain and dyspnoea overnight and is now hypotensive and tachycardic with cool peripheries and a raised jugular venous pressure. He is thought to be in cardiogenic shock and is awaiting transfer to the intensive care unit.
Which is the most common postoperative cause of cardiogenic shock?
A Hypovolaemia
B Myocardial infarction
C Pericardial tamponade
D Pulmonary embolus
E Ventricular fibrillation

A

The most common postoperative cause of cardiogenic shock, especially in an elderly patient with a history of chest pain and dyspnea, is myocardial infarction.

Answer:

B. Myocardial infarction

Explanation:

Cardiogenic shock occurs when the heart is unable to pump enough blood to meet the body’s needs, often due to a significant impairment in cardiac function. In the postoperative setting, especially following major surgery such as a laparotomy, myocardial infarction (MI) is a leading cause of cardiogenic shock. The symptoms described—chest pain, dyspnea, hypotension, tachycardia, cool peripheries, and raised jugular venous pressure—are all consistent with cardiogenic shock due to an MI.

•	Hypovolemia (A) is a common cause of shock but leads to hypovolemic shock, not cardiogenic shock. This is characterized by low central venous pressure, unlike the raised JVP seen here.
•	Pericardial tamponade (C) can cause cardiogenic shock but is less common postoperatively unless there is a specific reason to suspect it, such as chest trauma or recent cardiac surgery.
•	Pulmonary embolus (D) is a possible cause of postoperative shock but typically presents with signs such as acute shortness of breath, pleuritic chest pain, and signs of right heart strain rather than the more global heart failure picture suggested by raised JVP.
•	Ventricular fibrillation (E) would typically result in immediate loss of consciousness and is less likely to be described in a scenario where the patient remains conscious and hypotensive with raised JVP.

Therefore, myocardial infarction is the most likely cause of cardiogenic shock in this patient.

68
Q

A 23-year-old woman is under general anaesthesia for tonsillectomy. The anaesthetist requests that surgery does not commence, since the patient is tachycardic and hypotensive, and the anaesthetist is concerned that the patient is in anaphylactic shock.
Which is the most common cause of anaphylaxis under anaesthesia?
A Antibiotics, e.g. co-amoxiclav
B Colloids, e.g. gelofusine
C Induction agent, e.g. thiopentone
D Latex, e.g. surgical gloves
E Muscle relaxants, e.g. suxamethonium

A

The most common cause of anaphylaxis under anaesthesia is muscle relaxants, e.g. suxamethonium.

Answer:

E. Muscle relaxants, e.g. suxamethonium

Explanation:

Anaphylaxis during anaesthesia is most frequently caused by muscle relaxants. These agents can trigger a severe allergic reaction, leading to symptoms such as tachycardia, hypotension, bronchospasm, and urticaria. Suxamethonium and other neuromuscular blocking agents are well-documented as common culprits for such reactions.

•	Antibiotics, e.g. co-amoxiclav (A): While antibiotics can cause anaphylaxis, they are less commonly the cause compared to muscle relaxants in the setting of anaesthesia.
•	Colloids, e.g. gelofusine (B): Colloids can cause anaphylaxis, but they are not the most common cause under anaesthesia.
•	Induction agent, e.g. thiopentone (C): Induction agents can cause allergic reactions, but they are also less commonly the cause compared to muscle relaxants.
•	Latex, e.g. surgical gloves (D): Latex allergy is a concern, especially in patients with a history of latex sensitivity, but muscle relaxants are more frequently implicated in anaphylactic reactions during anaesthesia.

Therefore, muscle relaxants like suxamethonium are the most common cause of anaphylaxis in the anaesthetic setting.

69
Q

A 67-year-old man presents to the emergency department with abdominal pain, jaundice and pyrexia suggestive of acute cholangitis. He is tachycardic and hypotensive. The consultant asks for the Surviving Sepsis Resuscitation bundle to be commenced.
Which of the following is the most appropriate initial management?

A Administer broad spectrum antibiotic within 24 hours of admission
B Blood cultures are not essential if broad spectrum antibiotics are used
C Consider vasopressors if hypotension does not respond to minimum 20 mL/kg initial fluid resuscitation
D Maintain central venous pressure >12 mmHg
E Measure lactate on an arterial blood sample

A

The most appropriate initial management according to the Surviving Sepsis Resuscitation bundle is to consider vasopressors if hypotension does not respond to a minimum 20 mL/kg initial fluid resuscitation.

Answer:

C. Consider vasopressors if hypotension does not respond to minimum 20 mL/kg initial fluid resuscitation

Explanation:

The Surviving Sepsis Campaign guidelines recommend early identification and aggressive management of sepsis and septic shock, including:

•	Administer broad spectrum antibiotic within 24 hours of admission (A): Antibiotics should be administered much sooner, ideally within the first hour of recognizing sepsis.
•	Blood cultures are not essential if broad spectrum antibiotics are used (B): Blood cultures are essential and should be obtained before administering antibiotics, if this does not cause significant delay, to identify the causative organism.
•	Consider vasopressors if hypotension does not respond to minimum 20 mL/kg initial fluid resuscitation (C): This is correct. If the patient remains hypotensive after adequate fluid resuscitation, vasopressors should be considered to maintain adequate mean arterial pressure.
•	Maintain central venous pressure >12 mmHg (D): The current guidelines focus more on dynamic assessment of fluid responsiveness rather than static targets like central venous pressure.
•	Measure lactate on an arterial blood sample (E): Lactate measurement is essential, but it can be measured on either arterial or venous blood samples.

Therefore, considering vasopressors if hypotension does not respond to an initial fluid resuscitation of at least 20 mL/kg is the most appropriate initial management step according to the Surviving Sepsis Resuscitation bundle.

70
Q

A 70-year-old man on surgical high-dependency unit has been admitted with a 2-day history of vomiting and upper abdominal pain. He is hypotensive and hypovolaemic, he has cool peripheries and increased capillary refill. His hypotension is being treated by increasing cardiac preload with a fluid challenge of 500 mL crystalloid solution.
Which of the following most accurately describes this patient’s cardiac preload?
A Central venous pressure
B End diastolic ventricular volume
C Pulmonary artery occlusion pressure
D Ventricular myocyte length
E Ventricular myocyte tension

A

The most accurate description of cardiac preload is end diastolic ventricular volume.

Answer:

B. End diastolic ventricular volume

Explanation:

Preload refers to the initial stretching of the cardiac myocytes prior to contraction, which is related to the volume of blood filling the ventricles at the end of diastole. This volume directly influences the cardiac muscle fibers’ length and tension before systole, determining the stroke volume and overall cardiac output according to the Frank-Starling law of the heart.

•	Central venous pressure (A) is often used as a surrogate measure of preload, but it is not a direct measurement of the actual volume in the ventricles.
•	Pulmonary artery occlusion pressure (C), also known as wedge pressure, is another surrogate that estimates left ventricular preload, but it is still an indirect measure.
•	Ventricular myocyte length (D) and ventricular myocyte tension (E) are related to the concept of preload, as they describe the physical properties of the cardiac muscle fibers, but they do not directly quantify the volume of blood in the ventricles.

Therefore, the end diastolic ventricular volume most accurately describes cardiac preload.

71
Q

A 65-year-old man on the surgical high-dependency unit has been complaining of chest pain for the past hour. Cardiac ischaemia is suspected since the patient has a history of angina and his ECG shows ST depression in the lateral leads.
What is the main determinant of his coronary blood flow?
A Autonomic nervous system
B Blood viscosity
C Coronary artery diameter
D Diastolic blood pressure
E Myocardial oxygen demand

A

The main determinant of coronary blood flow is the coronary artery diameter.

Answer:

C. Coronary artery diameter

Explanation:

Coronary blood flow is significantly influenced by the diameter of the coronary arteries. Any reduction in the diameter, such as from atherosclerotic plaque, spasm, or other factors, can critically reduce blood flow to the myocardium, leading to ischemia. The diameter of the coronary arteries can change in response to various stimuli, including metabolic demands, autoregulation, and endothelial function.

•	Autonomic nervous system (A): While it can influence heart rate and contractility, its direct impact on coronary blood flow is secondary to the effects on coronary artery diameter and myocardial oxygen demand.
•	Blood viscosity (B): It has an effect on blood flow, but it is not the primary determinant of coronary blood flow.
•	Diastolic blood pressure (D): Important for coronary perfusion pressure, especially during diastole, but it is still secondary to the impact of coronary artery diameter.
•	Myocardial oxygen demand (E): Increases in myocardial oxygen demand lead to compensatory increases in coronary blood flow, but this is mediated by changes in coronary artery diameter.

Therefore, the main determinant of coronary blood flow in this context is the coronary artery diameter.

72
Q

A 58-year-old man on the postoperative surgical ward has hypotension. He underwent a laparoscopic cholecystectomy earlier today. The patient has a history of ischaemic heart disease. To treat the hypotension his cardiac output will be optimised.
To increase cardiac output with least impact on myocardial oxygen demand which of the following physiological variables should be optimised?
A Afterload
B Contractility
C Heart rate
D Preload
E Serum calcium

A

To increase cardiac output with the least impact on myocardial oxygen demand, preload should be optimized.

Answer:

D. Preload

Explanation:

Optimizing preload improves cardiac output by increasing the volume of blood returning to the heart, thereby enhancing ventricular filling and stroke volume according to the Frank-Starling mechanism. This approach increases cardiac output without significantly increasing myocardial oxygen demand, as it relies on the heart’s inherent ability to pump more effectively when more blood fills the ventricles.

•	Afterload (A): Decreasing afterload can improve cardiac output, but manipulating it often involves using vasodilators, which may have other systemic effects. Increasing afterload would increase myocardial oxygen demand.
•	Contractility (B): Enhancing contractility with inotropes increases cardiac output but also significantly raises myocardial oxygen demand.
•	Heart rate (C): Increasing heart rate can improve cardiac output, but it also significantly increases myocardial oxygen demand and may reduce diastolic filling time, which can be detrimental.
•	Serum calcium (E): While calcium is essential for muscle contraction, manipulating serum calcium levels to affect cardiac output is not a common or practical approach.

Therefore, optimizing preload by ensuring adequate fluid status and venous return is the best approach to increase cardiac output with the least impact on myocardial oxygen demand.

73
Q

A 38-year-old woman underwent a laparoscopic biopsy of a vascular lesion. The biopsy results are being reviewed. On the histopathology report, there is no mention of the site of the biopsy, however, the biopsy is found to contain a large proportion of elastic tissue.
Based on this report, where is the elastic tissue most likely to originate from?
A Aorta
B Arteries
C Arterioles
D Veins
E Vena cava

A

The histopathology report indicating a large proportion of elastic tissue suggests that the tissue is most likely to originate from the aorta.

Answer:

A. Aorta

Explanation:

The aorta is characterized by a high content of elastic tissue, which allows it to withstand and dampen the pulsatile output of the heart. The elasticity of the aorta helps maintain continuous blood flow during diastole by allowing the vessel to stretch and recoil. This feature is more pronounced in the aorta than in other blood vessels.

•	Arteries (B): While arteries contain elastic tissue, the proportion is less than that found in the aorta.
•	Arterioles (C): Arterioles have less elastic tissue and more smooth muscle relative to their size.
•	Veins (D): Veins contain some elastic tissue, but it is much less than in the arteries and particularly the aorta.
•	Vena cava (E): The vena cava has some elastic tissue, but again, not as much as the aorta.

Given the significant amount of elastic tissue described in the biopsy, the aorta is the most likely source.

74
Q

A 56-year-old man is being assessed for repair of a large aortic aneurysm. As part of his assessment he is going to undergo cardiopulmonary exercise testing.
Which organ receives the greatest proportion of cardiac output before the start of the test?
A Brain
B Heart
C Liver
D Kidneys
E Skeletal muscle

A

Before the start of cardiopulmonary exercise testing, the organ that receives the greatest proportion of cardiac output is the liver.

Answer:

C. Liver

Explanation:

At rest, the liver receives the largest proportion of cardiac output compared to other organs. The liver is highly vascular and requires a substantial blood supply to perform its various metabolic, detoxification, and synthetic functions.

•	Brain (A): The brain receives a significant portion of cardiac output, approximately 15%, but this is less than what the liver receives.
•	Heart (B): The heart receives about 5% of the cardiac output, primarily to supply the myocardium via the coronary arteries.
•	Liver (C): The liver receives about 25-30% of the cardiac output, primarily through the hepatic artery and portal vein.
•	Kidneys (D): The kidneys receive around 20% of the cardiac output to facilitate filtration and urine formation.
•	Skeletal muscle (E): At rest, skeletal muscle receives a smaller proportion of cardiac output compared to the liver. However, during exercise, blood flow to skeletal muscles increases significantly.

Therefore, before the start of the test, the liver receives the greatest proportion of cardiac output.

75
Q

A 65-year-old woman on the surgical high-dependency unit is peri-arrest. Her heart rate is 30 beats per minute, SpO2 of 80% and the monitor is unable to measure a systolic blood pressure. High flow oxygen and IV fluids are being administered. The house officer asks if IV adrenaline should be given.
Which one of the following adrenoreceptors causes vasoconstriction by adrenaline?
A α1
B α2
C β1
D β2
E β3

A

The adrenoreceptor that causes vasoconstriction when stimulated by adrenaline is the α1 receptor.

Answer:

A. α1

Explanation:

Adrenaline (epinephrine) acts on various adrenergic receptors in the body, but vasoconstriction is primarily mediated by the stimulation of α1 receptors, which are located on the vascular smooth muscle cells. Activation of these receptors causes vasoconstriction, leading to an increase in vascular resistance and, consequently, blood pressure. This can be particularly useful in a peri-arrest situation to help maintain perfusion to vital organs.

•	α2 (B): These receptors are involved in feedback inhibition of norepinephrine release and have some vasoconstrictive effects, but their primary role is not direct vasoconstriction in response to adrenaline.
•	β1 (C): These receptors are primarily located in the heart and increase heart rate and contractility when stimulated by adrenaline.
•	β2 (D): These receptors cause vasodilation and bronchodilation when stimulated by adrenaline.
•	β3 (E): These receptors are involved in lipolysis and have a minimal role in vasoconstriction.

Therefore, the α1 receptors are the primary receptors responsible for vasoconstriction by adrenaline.

76
Q

A 21-year-old man is brought in by ambulance to the emergency department with an abdominal stab wound. He is in class 3 hypovolaemic shock and is being resuscitated by the emergency team. The carotid baroreceptors are some of the main sensors of intravascular volume status.
What is the afferent nerve supply of the carotid baroreceptors?
A Glossopharyngeal nerve
B Phrenic nerve
C Recurrent laryngeal nerve
D Sympathetic chain
E Vagus nerve

A

The afferent nerve supply of the carotid baroreceptors is the glossopharyngeal nerve.

Answer:

A. Glossopharyngeal nerve

Explanation:

The carotid baroreceptors, located in the carotid sinus, play a crucial role in sensing changes in blood pressure. The afferent signals from these baroreceptors are carried by the glossopharyngeal nerve (cranial nerve IX) to the brainstem, where they help regulate cardiovascular responses to maintain blood pressure.

•	Glossopharyngeal nerve (A): The correct answer, as it carries afferent signals from the carotid baroreceptors.
•	Phrenic nerve (B): This nerve primarily innervates the diaphragm and is not involved in baroreceptor signaling.
•	Recurrent laryngeal nerve (C): A branch of the vagus nerve, it innervates the muscles of the larynx but is not involved in baroreceptor signaling.
•	Sympathetic chain (D): Involved in the autonomic regulation of various body functions, but it does not carry afferent signals from baroreceptors.
•	Vagus nerve (E): While the vagus nerve carries afferent signals from the aortic baroreceptors, it is the glossopharyngeal nerve that carries afferent signals from the carotid baroreceptors.

Therefore, the glossopharyngeal nerve is responsible for transmitting the afferent signals from the carotid baroreceptors.

The neurohumoral response to acute hypovolaemia is complex and can be divided into an early phase where maintenance of tissue perfusion is the priority and a later phase aimed at restoring a normal circulating volume.
Carotid and aortic baroreceptors are stretch receptors that are stimulated by increased distention of the vessel. Following acute blood loss venous return decreases and due to the Frank–Starling mechanism blood pressure falls. Decreased intra-arterial pressure causes less vessel wall stretch and hence reduced activation of the baroreceptors. Afferent neurones from the carotid baroreceptors (also known as the carotid sinuses) travel in the glossopharyngeal nerves and from the aortic baroreceptors via the vagus. These afferent neurones synapse with cardiovascular control centres in the brainstem. Decreased firing of these afferent neurones
will lead to inhibition of central parasympathetic outflow and increased central sympathetic outflow. Increased sympathetic outflow causes vasoconstriction and an increase in heart rate and myocardial contractility thereby maintaining perfusion to vital organs.
Other components of the early phase include an increase in serum levels of antidiuretic hormone (ADH), causing retention of water and vasoconstriction (ADH is also known as vasopressin). The renin–angiotensin–aldosterone system (RAAS) is also activated causing vasoconstriction and sodium and water retention. Atrial natriuretic peptide secretion is inhibited and together with ADH and the RAAS
this leads to restoration of circulating volume. Increased erythropoiesis returns haemoglobin levels to normal over a few weeks.
The phrenic nerves supply motor function to the diaphragm and sensation to the mediastinal pleura and most of the diaphragm. The recurrent laryngeal nerve supplies the intrinsic muscles of the larynx (except cricothyroid) and sensation below the vocal cords.

77
Q

A 20-year-old healthy female college student is caught in a bomb blast and sustains an injury to her right leg. There is skin loss with abrasions, and a wood splinter hidden deep beneath a laceration on the outside of her thigh.
What is the most likely cause for the delay in wound healing?
A Diabetes mellitus
B Foreign body
C Malnutrition
D Site of wound
E Smoking

A

In this scenario, the most likely cause for the delay in wound healing is the presence of a foreign body.

Answer:

B. Foreign body

Explanation:

Foreign bodies, such as the wood splinter mentioned, can significantly impede the wound healing process. They can cause persistent inflammation, infection, and mechanical irritation, all of which can delay healing. Removing the foreign body is essential to allow the wound to heal properly.

•	Diabetes mellitus (A): While diabetes can delay wound healing, there is no indication that the patient has diabetes.
•	Malnutrition (C): Malnutrition can affect wound healing, but there is no evidence provided that the patient is malnourished.
•	Site of wound (D): The location of the wound can influence healing, but it is less likely to be a primary factor compared to the presence of a foreign body.
•	Smoking (E): Smoking can impair wound healing due to its effects on blood flow and tissue oxygenation, but there is no information suggesting that the patient smokes.

Therefore, the presence of the wood splinter (foreign body) is the most likely cause for the delay in wound healing in this case.

78
Q

36-year-old man underwent an emergency conventional open appendicectomy for acute appendicitis and was discharged after 48 hours. He presented to the emergency department 3 days later with a high swinging pyrexia and severe pain in the stitch line. There were signs of inflammation and a yellowish discharge from the wound.
Which of the following is the most appropriate management of this patient?
A CT scan of the abdomen
B Open the wound on the ward
C Re-exploration in operating theatre
D Start antibiotics
E Start intravenous fluids and antipyretics

A

In the case of a postoperative wound infection presenting with signs of inflammation, yellowish discharge, and a high swinging pyrexia, the most appropriate initial management is to open the wound on the ward.

Answer:

B. Open the wound on the ward

Explanation:

Opening the wound on the ward allows for immediate drainage of any purulent material and helps relieve the infection. This step is essential for proper wound management and to control the infection locally. Once the wound is opened and drained, further steps such as wound culture, antibiotics, and other supportive measures can be considered based on the clinical findings.

•	CT scan of the abdomen (A): While imaging might be useful in certain scenarios to rule out deeper infections or abscesses, the initial step in managing a clearly infected surgical wound is to address the infection directly.
•	Re-exploration in the operating theatre (C): This is generally reserved for cases where there is suspicion of intra-abdominal complications that cannot be managed with simple drainage.
•	Start antibiotics (D): Antibiotics may be necessary, but they should be started after obtaining wound cultures and addressing the local infection through drainage.
•	Start intravenous fluids and antipyretics (E): These supportive measures can help manage symptoms but do not address the underlying source of infection.

Therefore, opening the wound on the ward for drainage is the most appropriate initial management in this situation.

79
Q

A 45-year-old woman presents with bouts of right upper abdominal pain associated with fatty meals. She is diagnosed as has having multiple gallstones with chronic cholecystitis and undergoes an elective laparoscopic cholecystectomy with a conventional four-port approach.
Which of the following terms best describes the wounds created for the ports?
A Clean
B Clean contaminated
C Contaminated
D Dirty
E Dirty infected

A

The wounds created for the ports during an elective laparoscopic cholecystectomy are best described as clean contaminated.

Answer:

B. Clean contaminated

Explanation:

Surgical wounds are classified based on their risk of infection:

•	Clean (A): These wounds are created under sterile conditions without entering the respiratory, alimentary, or genitourinary tracts. An example is a hernia repair.
•	Clean contaminated (B): These wounds involve controlled entry into the respiratory, alimentary, or genitourinary tracts under elective and controlled conditions without significant spillage or contamination. Laparoscopic cholecystectomy fits this category as it involves entering the biliary tract, which is a part of the alimentary system.
•	Contaminated (C): These wounds involve significant spillage from the gastrointestinal tract, major breaks in sterile technique, or traumatic wounds with non-purulent inflammation.
•	Dirty (D) and Dirty infected (E): These wounds involve existing infection or perforated viscera, often with purulent discharge.

Therefore, the wounds created for the ports during a laparoscopic cholecystectomy are considered clean contaminated because the procedure involves controlled entry into the biliary tract.

80
Q

A 50-year-old man, a heavy smoker, suffers from long-standing gastro-oesophageal reflux disease. Histology of the lower one-third of the oesophagus shows a Barrett’s oesophagus with an associated mucosal abnormality.
Which of the following is the most likely abnormality?
A Accretionary growth
B Differentiation
C Dysplasia
D Heterotopia
E Metaplasia

A

The most likely histological abnormality associated with Barrett’s oesophagus in this patient is dysplasia.

Answer:

C. Dysplasia

Explanation:

Barrett’s oesophagus is a condition in which the normal squamous epithelium of the lower oesophagus is replaced by columnar epithelium, which is more typical of the intestinal lining. This process is known as metaplasia. However, the question specifies an “associated mucosal abnormality,” which is most likely referring to dysplasia in the context of Barrett’s oesophagus. Dysplasia represents a precancerous change and indicates abnormal epithelial cell growth and differentiation, which can progress to adenocarcinoma if not monitored and managed appropriately.

•	Accretionary growth (A): This term is not typically used in the context of Barrett’s oesophagus.
•	Differentiation (B): Differentiation refers to the process by which cells become specialized, but it is not specific to the abnormal changes seen in Barrett’s oesophagus.
•	Dysplasia (C): Dysplasia indicates abnormal cell growth and is a precancerous condition often seen in Barrett’s oesophagus.
•	Heterotopia (D): Heterotopia refers to the presence of normal tissue in an abnormal location, which is not the primary concern in Barrett’s oesophagus.
•	Metaplasia (E): Metaplasia is the initial change in Barrett’s oesophagus where the squamous epithelium is replaced by columnar epithelium. However, the “associated mucosal abnormality” likely refers to dysplasia rather than the initial metaplastic change.

Therefore, dysplasia is the most likely associated mucosal abnormality in a patient with Barrett’s oesophagus.

81
Q

A 30-year-old man recently arrived from North Africa to the UK presents with haematuria. Cystoscopy reveals the presence of bilharzial nodules with papillomas. Histology does not show a carcinoma but does show a mucosal abnormality.
Which of the following is the most likely abnormality in the mucosa?
A Dysplasia
B Granuloma
C Metaplasia
D Multiplicative growth
E Pseudotubercles

A

In a patient presenting with haematuria and bilharzial nodules with papillomas, the most likely mucosal abnormality seen on histology would be a granuloma.

Answer:

B. Granuloma

Explanation:

Schistosomiasis (bilharzia) is caused by parasitic worms, and it is prevalent in North Africa. The presence of bilharzial nodules with papillomas in the bladder is indicative of chronic infection with Schistosoma haematobium. The body’s immune response to the eggs of the parasite often results in the formation of granulomas. These granulomas are characterized by collections of immune cells that form as the body attempts to wall off and contain the parasitic eggs.

•	Dysplasia (A): Dysplasia refers to abnormal growth and development of cells, which is a precancerous condition. However, it is not the typical response to schistosomiasis.
•	Granuloma (B): Granulomas are the characteristic histological finding in chronic schistosomiasis due to the immune response to the parasite’s eggs.
•	Metaplasia (C): Metaplasia refers to a change in the type of epithelium, which is not the primary feature in schistosomiasis.
•	Multiplicative growth (D): This term is not typically used in the context of schistosomiasis-related changes.
•	Pseudotubercles (E): Pseudotubercles are not a common finding in schistosomiasis. They are usually associated with other granulomatous diseases.

Therefore, the most likely mucosal abnormality in this patient with bilharzial nodules and papillomas is the formation of granulomas.

82
Q

A 65-year-old woman, on medical treatment for ulcerative colitis for 10 years, has been on annual colonoscopic surveillance. This is combined with random colonic biopsies.
Which of the following changes is the pathologist looking for in the biopsies?
A Auxetic growth
B Dysplasia
C Hyperplasia
D Metaplasia
E Regeneration

A

In a patient with a long history of ulcerative colitis undergoing annual colonoscopic surveillance with random colonic biopsies, the pathologist is primarily looking for dysplasia.

Answer:

B. Dysplasia

Explanation:

Patients with long-standing ulcerative colitis are at an increased risk of developing colorectal cancer. Dysplasia refers to the presence of abnormal cells that have the potential to progress to cancer. Monitoring for dysplasia through regular surveillance biopsies is critical in managing these patients, as the detection of dysplasia can prompt early intervention to prevent progression to invasive carcinoma.

•	Auxetic growth (A): This term is not typically used in the context of pathological changes in ulcerative colitis.
•	Dysplasia (B): Dysplasia indicates abnormal, precancerous cell growth and is the primary concern in surveillance of patients with ulcerative colitis.
•	Hyperplasia (C): Hyperplasia refers to an increase in the number of normal cells, which is not specifically monitored in this context.
•	Metaplasia (D): Metaplasia refers to a change in cell type, which is not the main focus of surveillance in ulcerative colitis.
•	Regeneration (E): Regeneration refers to the repair and replacement of tissues, which is a normal process and not a pathological change looked for in this context.

Therefore, the pathologist is looking for dysplasia in the random colonic biopsies taken during surveillance colonoscopies in this patient with long-standing ulcerative colitis.

83
Q

A 40-year-old woman sustained a blunt injury to her breast as a result of a road traffic accident where the steering wheel struck her chest wall. Two weeks later presents to a surgeon with a breast lump at the site of the injury.
Which of the following is the most likely diagnosis?
A Apoptosis
B Caseous necrosis
C Coagulative necrosis
D Colliquative necrosis
E Fat necrosis

A

The most likely diagnosis for a breast lump developing two weeks after a blunt injury to the breast is fat necrosis.

Answer:

E. Fat necrosis

Explanation:

Fat necrosis is a benign inflammatory response to trauma or surgery in the fatty tissue of the breast. It often presents as a lump, which can be tender and sometimes mimic malignancy, but it is non-cancerous. The history of trauma, such as a blunt injury from a road traffic accident, makes fat necrosis the most likely diagnosis in this scenario.

•	Apoptosis (A): Apoptosis is a programmed cell death process and does not typically result in a palpable lump.
•	Caseous necrosis (B): Caseous necrosis is associated with granulomatous infections like tuberculosis and is not typical in the context of trauma.
•	Coagulative necrosis (C): Coagulative necrosis is usually seen in ischemic injury to solid organs, such as the heart and kidneys, and not in the breast following trauma.
•	Colliquative necrosis (D): Also known as liquefactive necrosis, it is typically associated with brain tissue and abscess formation, not with trauma to the breast.

Therefore, the presence of a breast lump following a blunt injury is most consistent with fat necrosis.

84
Q

A 17-year-old man is admitted to hospital following a road accident. His spleen is removed because it is ruptured.
What role of the spleen will be lost with its removal, that may affect the patient’s future health?
A Deletion of immature autoreactive T cells
B Major site of insulin producing cells
C Primary lymphoid organ where immature lymphocytes arise
D Removal of encapsulated bacteria by phagocytic mechanisms
E Responds to the pituitary gland to produce thyroxine

A

The role of the spleen that will be lost with its removal, which may affect the patient’s future health, is removal of encapsulated bacteria by phagocytic mechanisms.

Answer:

D. Removal of encapsulated bacteria by phagocytic mechanisms

Explanation:

The spleen plays a crucial role in the immune system, particularly in filtering blood and removing pathogens, including encapsulated bacteria (such as Streptococcus pneumoniae, Haemophilus influenzae, and Neisseria meningitidis) via its phagocytic cells. Splenectomy (removal of the spleen) significantly reduces the body’s ability to fight off these types of bacteria, increasing the risk of severe infections. This condition is known as overwhelming post-splenectomy infection (OPSI).

•	Deletion of immature autoreactive T cells (A): This function is primarily performed by the thymus, not the spleen.
•	Major site of insulin-producing cells (B): Insulin is produced by the pancreas, not the spleen.
•	Primary lymphoid organ where immature lymphocytes arise (C): The bone marrow and thymus are the primary lymphoid organs responsible for the generation and maturation of lymphocytes.
•	Responds to the pituitary gland to produce thyroxine (E): The thyroid gland produces thyroxine in response to signals from the pituitary gland, not the spleen.

Therefore, the critical role lost with the removal of the spleen is its ability to remove encapsulated bacteria by phagocytic mechanisms, impacting the patient’s immune function and increasing the risk of infection.

85
Q

A 55-year-old man scheduled for surgery was found to be HIV positive at pre-assessment.
What is the most likely effect of this infection on his immune response to infection?
A Ability to phagocytose and kill bacteria will be lost
B Activation of an effective adaptive immune response will be impaired
C Activation of the alternative complement pathway will be prevented
D None. HIV infection does not affect the immune response
E Innate immune defences will be impaired

A

The most likely effect of HIV infection on his immune response to infection is that activation of an effective adaptive immune response will be impaired.

Answer:

B. Activation of an effective adaptive immune response will be impaired

Explanation:

HIV (Human Immunodeficiency Virus) primarily targets CD4+ T cells, which are crucial for the activation and coordination of the adaptive immune response. The depletion of these cells leads to a weakened immune system, reducing the body’s ability to mount an effective response to infections and increasing susceptibility to opportunistic infections and certain cancers.

•	Ability to phagocytose and kill bacteria will be lost (A): HIV does not directly impair the function of phagocytes (such as macrophages and neutrophils) initially; however, over time, the overall immune dysfunction can indirectly affect phagocytic activity.
•	Activation of the alternative complement pathway will be prevented (C): The complement pathways, including the alternative pathway, are part of the innate immune system and are not directly affected by HIV infection.
•	None. HIV infection does not affect the immune response (D): This is incorrect; HIV significantly impairs the immune system, particularly the adaptive immune response.
•	Innate immune defences will be impaired (E): While HIV primarily affects the adaptive immune system, chronic HIV infection and the resultant immune dysfunction can indirectly impair innate immune responses over time.

Therefore, the most significant effect of HIV infection is the impairment of the adaptive immune response, due to the depletion of CD4+ T cells.

86
Q

A 60-year-old man with septicaemia due to an antibiotic resistant infection was admitted to hospital. In septicaemia, uncontrolled activation of inflammatory cytokines can lead to acute respiratory distress syndrome and multiorgan failure.
What are cytokines?
A Antigen-specific molecules produced by B lymphocytes
B Part of an enzyme cascade activated by immune complexes
C Small signalling proteins that regulate immune activation
D Subtype of cytotoxic T cells
E Ultracellular enzymes which kill bacteria within phagocytic cells

A

Cytokines are small signalling proteins that regulate immune activation.

Answer:

C. Small signalling proteins that regulate immune activation

Explanation:

Cytokines are a broad category of small proteins that are important in cell signaling. They are secreted by various cells, particularly those of the immune system, and play crucial roles in regulating the immune response, inflammation, and hematopoiesis. In the context of septicaemia (sepsis), an uncontrolled release of pro-inflammatory cytokines can lead to systemic inflammation, acute respiratory distress syndrome (ARDS), and multiorgan failure.

•	Antigen-specific molecules produced by B lymphocytes (A): This describes antibodies, not cytokines.
•	Part of an enzyme cascade activated by immune complexes (B): This describes components of the complement system, not cytokines.
•	Subtype of cytotoxic T cells (D): This describes a type of T cell, not cytokines.
•	Ultracellular enzymes which kill bacteria within phagocytic cells (E): This describes lysosomal enzymes, not cytokines.

Therefore, cytokines are the small signaling proteins that regulate immune activation and play a critical role in the immune response and inflammation.

87
Q

A 70-year-old woman is admitted for hip replacement. She had suffered for many years from chronic rheumatoid arthritis, an autoimmune disease with progressive joint destruction.
What is the most likely mechanism causing autoimmune disease?

A Body mounts an immune response against itself
B Chronic activation of allergen specific IgE sensitised mast cells initiates the disease
C Deficient immune response due to the action of an external agent
D Genetic defect causing a deranged immune response
E T cell activation is inhibited

A

The most likely mechanism causing autoimmune disease is that the body mounts an immune response against itself.

Answer:

A. Body mounts an immune response against itself

Explanation:

Autoimmune diseases occur when the body’s immune system mistakenly attacks its own tissues, recognizing them as foreign. This immune response can lead to chronic inflammation and progressive tissue damage, as seen in conditions like rheumatoid arthritis.

•	Chronic activation of allergen specific IgE sensitised mast cells initiates the disease (B): This describes the mechanism of allergic reactions, not autoimmune diseases.
•	Deficient immune response due to the action of an external agent (C): This could describe immunodeficiency, not autoimmunity.
•	Genetic defect causing a deranged immune response (D): While genetic factors can contribute to the development of autoimmune diseases, the primary mechanism is the immune system attacking self-tissues.
•	T cell activation is inhibited (E): This would describe a scenario of immunosuppression, not autoimmunity.

Therefore, autoimmune diseases are primarily caused by the body mounting an immune response against its own tissues.

88
Q

A 21-year-old woman is admitted to hospital for removal of wisdom teeth. Investigations reveal that she has high levels of circulating IgM antibody.
What statement below best describes IgM antibodies?
A Antibodies that can survive on mucosal surfaces
B Antibodies which bind to mast cells during allergic reactions
C Antibodies which can confer protection to newborn infants
D Long-lived antibodies that provide long-term protection against infection
E Short-lived antibodies produced during the early stages of ongoing infection

A

IgM antibodies are best described as short-lived antibodies produced during the early stages of ongoing infection.

Answer:

E. Short-lived antibodies produced during the early stages of ongoing infection

Explanation:

IgM is the first antibody isotype produced in response to an infection. It is effective at forming antigen-antibody complexes and is crucial in the initial stages of the immune response. IgM antibodies are typically short-lived and are later replaced by other isotypes such as IgG.

•	Antibodies that can survive on mucosal surfaces (A): This describes IgA antibodies, which are present in mucosal areas and secretions.
•	Antibodies which bind to mast cells during allergic reactions (B): This describes IgE antibodies, which play a role in allergic responses.
•	Antibodies which can confer protection to newborn infants (C): This describes IgG antibodies, which can cross the placenta and provide passive immunity to the fetus.
•	Long-lived antibodies that provide long-term protection against infection (D): This describes IgG antibodies, which are responsible for long-term immunity.

Therefore, IgM antibodies are short-lived antibodies produced during the early stages of an ongoing infection.

89
Q

A 30-year-old man is going to work in Uganda for 6 months. As a precaution he has a Heaf (six-needle) test, and this gives a positive reaction.
What is the most likely immunological significance of this result?
A Determines exposure to Lyme disease
B Determines protective antibody responses to vector-borne infections
C Determines T cell responses to the tubercle bacillus
D Measures allergic responses to grass and flower pollens
E No immunological significance

A

The Heaf test, also known as the Mantoux test or tuberculin skin test (TST), is used to determine whether a person has been exposed to the bacterium Mycobacterium tuberculosis, which causes tuberculosis (TB). The test involves injecting a small amount of tuberculin purified protein derivative (PPD) into the skin and observing the reaction.

Given this information, the most likely immunological significance of a positive Heaf test result is:

C. Determines T cell responses to the tubercle bacillus

A positive reaction indicates that the person’s immune system has been exposed to TB bacteria and has mounted a T cell-mediated immune response. This does not necessarily mean the person has active tuberculosis, but it does suggest that they have been infected with the bacteria at some point. Further tests would be needed to determine if there is an active infection.

90
Q

A 70-year-old woman, admitted for a corneal graft, will not require immunosuppressive therapy after surgery because the anterior chamber of the eye is considered to be ‘immunologically privileged’.
What is the most likely immunological mechanism involved?
A Adaptive immune responses do not occur there.
B All immune cells are destroyed immediately upon arrival
C Beneficial responses are promoted while damaging ones are suppressed.
D Immune cells arrive but are prevented from entering
E Immune cells become functionally inactive on entering the site

A

The concept of “immune privilege” refers to certain sites in the body where immune responses are limited or regulated to prevent damage to delicate tissues. The anterior chamber of the eye is one such site. This privilege helps to protect the eye from inflammatory damage that could impair vision.

Given this context, the most likely immunological mechanism involved in why the woman will not require immunosuppressive therapy after a corneal graft is:

C. Beneficial responses are promoted while damaging ones are suppressed.

This mechanism explains how the eye can tolerate foreign tissue, such as a corneal graft, without initiating a damaging immune response. In immune-privileged sites, the immune system is selectively regulated to allow tolerance of antigens without causing harmful inflammation.

91
Q

A 35-year-old woman is having preoperative screening before laparoscopic cholecystectomy. She has had four children in the last 10 years, during which time she suffered from heavy periods. For many years she has been on oral iron preparations intermittently. A full blood count shows haemoglobin of 105 g/L, mean cell volume is low, reticulocyte count is normal, red blood cells are pale.
What is the most likely diagnosis?
A Macrocytic anaemia due to failure of DNA replication
B Macrocytic anaemia due to iron deficiency
C Microcytic anaemia due to failure of DNA replication
D Microcytic anaemia due to iron deficiency
E Normocytic anaemia due to bone marrow failure

A

Given the details provided:

•	Haemoglobin of 105 g/L, which indicates anemia (normal range for women is typically 120-160 g/L).
•	Mean cell volume (MCV) is low, indicating microcytosis.
•	Red blood cells are pale, suggesting hypochromia.
•	The patient has a history of heavy menstrual bleeding, which can lead to iron deficiency.
•	She has been on oral iron preparations intermittently, suggesting she has been treated for iron deficiency.

The most likely diagnosis is:

D. Microcytic anaemia due to iron deficiency

Iron deficiency anemia is characterized by low hemoglobin levels, low MCV (microcytosis), and hypochromic red blood cells. This is consistent with the patient’s history of heavy menstrual bleeding and intermittent iron supplementation.

92
Q

A 74-year-old woman complains of gradually increasing tiredness over the last 6–12 months. She is anaemic, with a haemoglobin level of 105 g/L. Mean red cell volume is above the normal range. The reticulocyte count is around the lower limit of normal.
What is the most likely diagnosis?
A Macrocytic anaemia due to deficiency of vitamin B2
B Macrocytic anaemia due to deficiency of vitamin B12
C Microcytic anaemia due to deficiency of vitamin B12
D Normocytic anaemia due to chronic liver failure
E Normocytic anaemia due to chronic renal failure

A

Given the details provided:

•	The patient is anemic with a hemoglobin level of 105 g/L.
•	The mean red cell volume (MCV) is above the normal range, indicating macrocytosis.
•	The reticulocyte count is around the lower limit of normal.

The most likely diagnosis is:

B. Macrocytic anaemia due to deficiency of vitamin B12

Macrocytic anemia is often caused by deficiencies in vitamin B12 or folate. Given the patient’s age and the gradually increasing tiredness, a deficiency in vitamin B12 is a likely cause. This type of anemia is characterized by larger than normal red blood cells and can be accompanied by neurological symptoms if left untreated.

93
Q

A 32-year-old man is being treated by peritoneal dialysis for chronic renal failure. His haemoglobin level is 105 g/L. His mean cell volume is normal and the red cells look normal microscopically.
Which of the following is the most effective management of his anaemia?
A Erythropoietin to stimulate his bone marrow
B Oral iron plus vitamin B12 to stimulate his bone marrow
C Oral iron to stimulate his bone marrow
D Parenteral iron and B12 to stimulate his bone marrow
E Regular transfusions of red cells

A

Given the patient’s clinical scenario:

•	The patient has chronic renal failure and is undergoing peritoneal dialysis.
•	His hemoglobin level is 105 g/L, indicating anemia.
•	His mean cell volume (MCV) is normal, and red cells look normal microscopically, suggesting normocytic anemia.

In chronic renal failure, anemia is commonly caused by a deficiency in erythropoietin, a hormone produced by the kidneys that stimulates red blood cell production in the bone marrow. The most effective treatment for anemia in patients with chronic renal failure is:

A. Erythropoietin to stimulate his bone marrow

Explanation:

1.	Erythropoietin Deficiency: In chronic renal failure, the kidneys fail to produce adequate amounts of erythropoietin, leading to reduced red blood cell production and anemia.
2.	Effective Treatment: Administering erythropoietin directly addresses the primary cause of anemia in these patients. It stimulates the bone marrow to produce more red blood cells.
3.	Other Options: While iron and vitamin B12 can be beneficial if deficiencies exist, they are not the primary issue in this scenario. Regular transfusions are not the first-line treatment due to risks such as iron overload and alloimmunization.

Thus, erythropoietin is the most effective and appropriate management for anemia in a patient with chronic renal failure undergoing peritoneal dialysis.

94
Q

A 6-year-old girl is awaiting surgery for cyanotic heart disease. Her cyanosis is central. Her haemoglobin is 165 g/L.
Which of the following is most likely to have caused her polycythaemia?
A Decreased circulating erythropoietin
B Direct stimulation of her red bone marrow by hypoxia
C Direct stimulation of her red bone marrow by a raised carbon dioxide level
D Frequent blood transfusions
E Increased circulating erythropoietin

A

The 6-year-old girl with cyanotic heart disease and central cyanosis has a hemoglobin level of 165 g/L, indicating polycythemia.

Given the context of cyanotic heart disease, the most likely cause of her polycythemia is:

E. Increased circulating erythropoietin

Explanation:

1.	Cyanotic Heart Disease: Cyanotic heart disease leads to chronic hypoxia, as the body is not receiving enough oxygen due to the heart’s inability to properly oxygenate the blood.
2.	Erythropoietin Response: The kidneys respond to chronic hypoxia by increasing the production of erythropoietin, a hormone that stimulates red blood cell production in the bone marrow.
3.	Polycythemia: Increased levels of erythropoietin lead to increased red blood cell production, resulting in polycythemia, which is a compensatory mechanism to improve oxygen delivery to tissues.

Incorrect Options:

•	A. Decreased circulating erythropoietin: This would lead to anemia, not polycythemia.
•	B. Direct stimulation of her red bone marrow by hypoxia: While hypoxia stimulates erythropoietin production, it is the increased erythropoietin that directly stimulates the bone marrow.
•	C. Direct stimulation of her red bone marrow by a raised carbon dioxide level: Elevated CO2 levels do not directly stimulate the bone marrow. It is the hypoxia that triggers erythropoietin production.
•	D. Frequent blood transfusions: This could cause polycythemia, but there is no indication in the scenario that the child is receiving frequent transfusions.

Thus, the increased circulating erythropoietin due to chronic hypoxia from cyanotic heart disease is the most likely cause of her polycythemia.

95
Q

A 60-year-old man is being managed as an outpatient on a chemotherapy regimen for non-Hodgkin lymphoma. He has developed a painful throat with irregular ulceration on his pharynx.
Which one of the following is most likely to have caused his throat problem?
A Acute depression of bone marrow function
B Acute lymphatic leukaemia
C Exacerbation of chronic myeloid leukaemia
D Throat infection with Clostridium difficile organisms
E Throat infection with MRSA organisms

A

Given the context of a 60-year-old man undergoing chemotherapy for non-Hodgkin lymphoma who has developed a painful throat with irregular ulceration on his pharynx, the most likely cause of his throat problem is:

A. Acute depression of bone marrow function

Explanation:

1.	Chemotherapy Side Effects: Chemotherapy for non-Hodgkin lymphoma can cause myelosuppression, which is a significant reduction in bone marrow activity. This results in decreased production of white blood cells, red blood cells, and platelets.
2.	Neutropenia and Infections: The reduction in white blood cells, particularly neutrophils, can lead to neutropenia, making the patient highly susceptible to infections, including those in the oral and pharyngeal regions.
3.	Mucosal Damage: Chemotherapy can also cause direct damage to the mucosal lining of the mouth and throat, leading to mucositis, which presents as painful ulcerations and inflammation.

Incorrect Options:

•	B. Acute lymphatic leukemia: This is a different condition and not directly related to the patient’s current chemotherapy for non-Hodgkin lymphoma.
•	C. Exacerbation of chronic myeloid leukemia: This is another unrelated condition and would not be a typical consequence of chemotherapy for non-Hodgkin lymphoma.
•	D. Throat infection with Clostridium difficile organisms: C. difficile primarily causes gastrointestinal infections, particularly colitis, not throat infections.
•	E. Throat infection with MRSA organisms: While possible, MRSA infections typically occur in hospitalized patients or those with invasive devices. The description suggests a more general immune suppression and susceptibility to infections due to chemotherapy.

Therefore, the most likely cause of his throat problem is the acute depression of bone marrow function resulting from his chemotherapy regimen.

96
Q

A 27-year-old man has been brought to hospital unconscious, with many superficial cuts and abrasions. Blood loss from these is much more extensive and persistent than would be expected. His personal details, previous medical history and possible medication are unknown at this stage.
Which of the following is the most likely cause of this man’s excessive bleeding?
A He completed, two weeks previously, a course of low molecular weight heparin to prevent deep venous thrombosis
B He is on warfarin treatment and his INR is 1.2
C He is on warfarin treatment and his INR is 0.8
D His factor VIII level is about 20% of the normal average
E His platelet count is slightly below the lower limit of the normal range

A

Given the context of a 27-year-old man with extensive and persistent blood loss from superficial cuts and abrasions, the most likely cause of this excessive bleeding is:

D. His factor VIII level is about 20% of the normal average

Explanation:

1.	Factor VIII Deficiency (Hemophilia A): A significantly reduced level of factor VIII (about 20% of the normal average) suggests a moderate form of Hemophilia A. Hemophilia A is a genetic disorder that impairs the body’s ability to make blood clots, leading to prolonged bleeding from even minor injuries.
2.	Excessive and Persistent Bleeding: This level of factor VIII deficiency would result in more extensive and prolonged bleeding from superficial cuts and abrasions than would normally be expected.

Incorrect Options:

•	A. He completed, two weeks previously, a course of low molecular weight heparin to prevent deep venous thrombosis: The effects of low molecular weight heparin typically do not persist for two weeks after cessation of therapy.
•	B. He is on warfarin treatment and his INR is 1.2: An INR of 1.2 is within the normal range (therapeutic range for warfarin treatment is typically 2-3), suggesting no significant anticoagulation effect.
•	C. He is on warfarin treatment and his INR is 0.8: An INR of 0.8 is below the normal range, indicating that his blood is more prone to clotting rather than excessive bleeding.
•	E. His platelet count is slightly below the lower limit of the normal range: A slightly low platelet count would not usually cause extensive and persistent bleeding from superficial cuts and abrasions. Severe thrombocytopenia (very low platelet count) would be required to cause such symptoms.

Therefore, a significantly low factor VIII level is the most likely cause of his excessive bleeding.

97
Q

A 50-year-old woman requires total parenteral feeding.
Which of the following is the most appropriate management to meet her 24-hour energy requirements?
A 5% dextrose and 10% amino acid solution
B 20% dextrose and 10% lipid
C 5% dextrose as the sole provider of energy
D 50% dextrose as the sole provider of energy
E 20% lipid as the sole provider of energ

A

Total parenteral nutrition (TPN) should provide a balanced mix of carbohydrates, proteins, and fats to meet the energy and nutrient requirements of a patient. The most appropriate management to meet the 24-hour energy requirements for a 50-year-old woman requiring total parenteral feeding would be:

B. 20% dextrose and 10% lipid

Explanation:

1.	Balanced Nutrient Supply: Total parenteral nutrition should include a balanced combination of macronutrients (carbohydrates, proteins, and fats) to adequately meet the patient’s energy needs and maintain metabolic balance.
2.	Energy Density: A combination of 20% dextrose and 10% lipid provides a substantial amount of energy. Dextrose (glucose) is a primary source of carbohydrate energy, while lipids provide a dense source of calories and essential fatty acids.
3.	Amino Acids: While not explicitly mentioned in option B, TPN typically also includes amino acids to meet the protein needs of the patient. It’s essential to include amino acids in TPN to prevent muscle wasting and promote tissue repair.

Incorrect Options:

•	A. 5% dextrose and 10% amino acid solution: This combination may not provide sufficient energy because 5% dextrose is a low concentration for carbohydrate energy, and amino acids are primarily for protein needs, not energy.
•	C. 5% dextrose as the sole provider of energy: This would not provide sufficient energy and lacks essential proteins and fats, leading to nutritional deficiencies.
•	D. 50% dextrose as the sole provider of energy: This would provide a high carbohydrate load, which could cause hyperglycemia and other metabolic complications. It also lacks proteins and fats.
•	E. 20% lipid as the sole provider of energy: Lipids alone cannot meet all nutritional requirements and may lead to complications if used as the sole energy source, such as essential fatty acid deficiency and insufficient carbohydrate and protein intake.

Therefore, the combination of 20% dextrose and 10% lipid is the most appropriate choice for meeting the 24-hour energy requirements of a patient requiring total parenteral feeding.

98
Q

A 60-year-old man with a long history of duodenal ulcer has the typical clinical features of gastric outlet obstruction with incessant non-bilious vomiting, visible gastric peristalsis and succussion splash. He has Chvostek’s sign (tapping in front of the ear causes spasm of the facial muscles).
Which of the following is most likely to have caused his spasm?
A Acidotic tetany from respiratory acidosis
B Acidotic tetany from respiratory alkalosis
C Alkalotic tetany from metabolic alkalosis
D Alkalotic tetany from respiratory acidosis
E Alkalotic tetany from respiratory alkalosis

A

The most likely cause of the spasm in a 60-year-old man with a long history of duodenal ulcer, presenting with gastric outlet obstruction and Chvostek’s sign, is:

C. Alkalotic tetany from metabolic alkalosis

Explanation:

1.	Gastric Outlet Obstruction: This condition leads to incessant vomiting, which results in the loss of gastric acid (hydrochloric acid).
2.	Metabolic Alkalosis: The loss of gastric acid through vomiting results in a decrease in hydrogen ions and an increase in bicarbonate ions, leading to metabolic alkalosis.
3.	Chvostek’s Sign: This is a clinical sign of hypocalcemia, which can be a result of metabolic alkalosis. In alkalosis, the ionized calcium levels in the blood decrease because calcium binds more to albumin when the pH is high, leading to symptoms like muscle spasms and tetany.
4.	Non-bilious Vomiting: Since the vomiting is non-bilious (does not contain bile), it suggests that the obstruction is above the level where bile enters the gastrointestinal tract (i.e., proximal to the duodenum).

Incorrect Options:

•	A. Acidotic tetany from respiratory acidosis: Respiratory acidosis results from hypoventilation and an increase in CO2 levels, leading to acidemia. This does not fit the clinical scenario of incessant vomiting.
•	B. Acidotic tetany from respiratory alkalosis: Respiratory alkalosis results from hyperventilation and a decrease in CO2 levels. It can cause tetany, but it does not match the clinical context of gastric outlet obstruction.
•	D. Alkalotic tetany from respiratory acidosis: This is a contradictory statement, as respiratory acidosis is an acidotic condition, not alkalotic.
•	E. Alkalotic tetany from respiratory alkalosis: Respiratory alkalosis is not related to vomiting and gastric outlet obstruction, which clearly indicate a gastrointestinal cause.

Therefore, the patient’s spasm is most likely due to alkalotic tetany from metabolic alkalosis, resulting from the loss of gastric acid due to incessant vomiting caused by gastric outlet obstruction.

99
Q

A 25-year-old woman, who does not look after her type 1 diabetes well, has been brought into the emergency department with diabetic ketoacidosis. She has a high anion gap acidosis.
Which of the following is most likely to have caused the acidosis?
A A decrease in bicarbonate concentration
B A decrease in chloride concentration
C An increase in potassium concentration
D An increase in sodium concentration
E Combined increase in sodium and decrease in chloride

A

The most likely cause of the acidosis in a 25-year-old woman with poorly managed type 1 diabetes who presents with diabetic ketoacidosis and a high anion gap acidosis is:

A. A decrease in bicarbonate concentration

Explanation:

1.	Diabetic Ketoacidosis (DKA): In DKA, the body breaks down fats into ketones due to the lack of insulin, leading to the accumulation of acidic ketone bodies in the blood, which causes metabolic acidosis.
2.	High Anion Gap Acidosis: This is characterized by an increase in unmeasured anions (such as ketones) in the blood, leading to a high anion gap. The anion gap is calculated as [Na+] - ([Cl-] + [HCO3-]).
3.	Decrease in Bicarbonate: The accumulation of ketones in DKA leads to a buffering response that consumes bicarbonate, resulting in a decreased bicarbonate concentration, which is a primary cause of the high anion gap acidosis seen in DKA.

Incorrect Options:

•	B. A decrease in chloride concentration: A decrease in chloride concentration would not directly cause a high anion gap acidosis. Chloride levels may decrease in some forms of metabolic alkalosis but are not the primary factor in DKA.
•	C. An increase in potassium concentration: While hyperkalemia can be seen in DKA due to shifts of potassium from cells to the blood, it is not the primary cause of the acidosis.
•	D. An increase in sodium concentration: An increase in sodium concentration does not cause acidosis and is not typically seen in DKA.
•	E. Combined increase in sodium and decrease in chloride: This scenario does not describe the pathophysiology of DKA and would not explain the high anion gap acidosis.

Therefore, the acidosis in this patient with diabetic ketoacidosis is most likely caused by a decrease in bicarbonate concentration.

100
Q

A 35-year-old patient presents with a painful swelling in the thenar eminence of the hand. The most likely anatomical structure involved is:
Options

A) Adductor pollicis muscle
B) Abductor pollicis longus tendon
C) Flexor pollicis brevis muscle
D) Radial artery

A

Correct answer: B) Abductor pollicis longus tendon
Explanation: The thenar eminence is a muscular prominence on the palmar surface of the hand, primarily composed of thumb muscles. The abductor pollicis longus tendon is prone to tenosynovitis or tendonitis, leading to pain and swelling in this area, commonly known as de Quervain’s tenosynovitis.
Additional Information

De Quervain’s tenosynovitis affects the tendons on the thumb side of your wrist.
Symptoms include pain and tenderness, especially when forming a fist, grasping or turning the wrist.
Treatment options range from physical therapy and injections to surgery.

101
Q

A surgical trainee is concerned about the operating theater’s infection control. Which measure is most effective in preventing surgical site infections?
Options

A) Preoperative skin scrub with antibacterial soap
B) Surgical team’s proper hand hygiene and double gloving
C) Patient warming and normothermia maintenance
D) All of the above

A

Correct answer: D) All of the above
Explanation: Preventing surgical site infections requires a multifaceted approach. While preoperative skin scrub with antibacterial soap (A) reduces skin flora, it’s just one aspect.
Proper hand hygiene and double gloving (B) minimize transmission from surgical staff.
Maintaining patient normothermia (C) reduces infection risk by promoting immune function, blood flow and oxygen delivery to tissues.
Additional measures include:
Antibiotic prophylaxis
Sterile equipment and supplies
Surgical technique and tissue handling
Postoperative wound care

102
Q

A 60-year-old male presents with a painless, slowly enlarging neck mass. Histopathology reveals lymphoid follicles with germinal centers. The most likely diagnosis is:
Options

A) Hodgkin lymphoma
B) Non-Hodgkin lymphoma (follicular)
C) Reactive lymphadenopathy
D) Papillary thyroid carcinoma

A

Correct answer: B) Non-Hodgkin lymphoma (follicular)
Explanation: Follicular lymphoma, a type of non-Hodgkin lymphoma (NHL), typically presents with painless lymphadenopathy (enlarged lymph nodes). Histopathology shows lymphoid follicles with germinal centers, characteristic of follicular lymphoma.
Key features:
Indolent course: Slow growth, often asymptomatic.
Follicular architecture: Germinal centers surrounded by mantle zones.
Immunohistochemistry: CD20+, CD10+, BCL2+.
Differential diagnoses:
Reactive lymphadenopathy (C): Typically resolves, lacks follicular pattern.
Hodgkin lymphoma (A): Presence of Reed-Sternberg cells.
Papillary thyroid carcinoma (D): Characteristic nuclear features, different clinical context.

103
Q

A 25-year-old male has a closed fracture of the distal radius with dorsal angulation. The appropriate initial management is:
Options

A) Immediate open reduction and internal fixation
B) Closed reduction under sedation
C) Application of a plaster cast
D) Traction and manipulation under anesthesia

A

Correct answer: B) Closed reduction under sedation
Explanation: Initial management for a closed distal radius fracture with dorsal angulation involves closed reduction to restore anatomical alignment, minimize complications and improve outcomes.
Steps for closed reduction:
Analgesia and sedation: Ensure patient comfort.
Assistant: Maintain traction.
Reduction maneuver: Apply gentle pressure to fracture fragments.
Post-reduction imaging: Confirm alignment.
Immobilization: Cast or splint application.
Indications for open reduction and internal fixation (ORIF) (Option A):
Unstable fractures
Failed closed reduction
Articular involvement
Neurovascular compromise
Other options:
Application of a plaster cast (C): Suitable after successful closed reduction.
Traction and manipulation under anesthesia (D): Typically reserved for complex or unstable fractures.

104
Q

Which of the following is a fundamental principle of surgical consent?
Options

A) Patient’s autonomy and informed decision
B) Surgeon’s expertise and preference
C) Hospital policy and protocol
D) Family’s approval and agreement

A

Answer and Explanation

Correct answer: A) Patient’s autonomy and informed decision
Explanation: Surgical consent prioritizes respecting patients’ autonomy, ensuring they make informed decisions. Key elements:
Capacity: Patients must understand implications.
Voluntariness: Free from coercion.
Information: Clear explanation of risks, benefits and alternatives.
Comprehension: Patients understand provided information.
Additional Principles

Respecting patient autonomy and dignity
Ensuring informed consent for all procedures
Documenting consent accurately
Considering cultural, linguistic and cognitive factors
Relevance to Surgical Practice

Informed consent promotes trust, accountability and patient-centered care. Breaches can lead to medico-legal consequences.

105
Q

A 40-year-old woman undergoes laparoscopic cholecystectomy for symptomatic gallstones. Which structure is most at risk during dissection of Calot’s triangle?
Options

A) Common bile duct
B) Hepatic artery
C) Portal vein
D) Right hepatic duct

A

Correct answer: A) Common bile duct
Explanation: Calot’s triangle, bounded by the liver edge, cystic duct and common hepatic duct, contains crucial structures. The common bile duct (CBD) is most vulnerable during dissection due to:
Anatomical variability: CBD location and course.
Proximity to cystic duct: Risk of misidentification.
Inflammation or fibrosis: Obscured anatomy.
Consequences of CBD injury:
Bile leakage: Peritonitis, abscesses.
Stricture formation: Cholestasis, jaundice.
Long-term complications: Liver damage, cirrhosis.
Prevention strategies:
Clear identification: Confirm anatomy.
Careful dissection: Minimize tissue damage.
Intraoperative cholangiography: Visualize bile ducts.

106
Q

A 65-year-old male presents with intermittent claudication. Investigations reveal significant stenosis of the superficial femoral artery. The best initial management is:
Options

A) Endovascular angioplasty and stenting
B) Surgical bypass with autologous vein
C) Medical therapy (antiplatelets, statins, lifestyle modification)
D) Thrombolysis

A

Correct answer: A) Endovascular angioplasty and stenting
Explanation: Endovascular intervention is the preferred initial management for intermittent claudication due to superficial femoral artery stenosis.
Rationale

Minimally invasive: Reduces recovery time and complications.
Effective symptom relief: Improves walking distance.
Low risk: Compared to open surgery.
Indications for Endovascular Intervention

Symptomatic stenosis: Intermittent claudication, critical limb ischemia.
Lesion characteristics: Short-segment stenosis, non-calcified.
Patient factors: Comorbidities, high surgical risk.
Alternative Options

Medical therapy (C): Best for mild symptoms or non-revascularizable patients.
Surgical bypass (B): Reserved for complex lesions, failed endovascular treatment or severe ischemia.
Thrombolysis (D): Suitable for acute limb ischemia.
Additional Considerations

Lifestyle modifications: Smoking cessation, exercise and healthy diet.
Pharmacological management: Antiplatelets, statins, ACE inhibitors.
Regular follow-up: Monitor disease progression, intervention effectiveness.

107
Q

A 50-year-old male presents with progressive weakness and sensory loss in the right arm. MRI reveals a spinal cord tumor at C5-C6. The appropriate surgical approach is:
Options

A) Anterior cervical discectomy and fusion (ACDF)
B) Posterior cervical laminectomy
C) Posterolateral approach (costotransversectomy)
D) Transoral odontoidectomy

A

Correct answer: B) Posterior cervical laminectomy
Explanation: For spinal cord tumors at C5-C6, posterior cervical laminectomy is often preferred.
Rationale

Direct access: Posterior approach provides straightforward access to spinal cord tumors.
Minimal disruption: Preserves anterior spinal stability.
Effective decompression: Relieves spinal cord compression.
Indications

Intraspinal tumors: Meningiomas, neurofibromas, schwannomas.
Spinal cord compression: Symptomatic cord impingement.
Posterior or lateral tumor location: Optimizes surgical exposure.
Considerations

Tumor histology: Influences surgical strategy.
Spinal stability: May require fusion.
Neurological status: Guides surgical urgency.
Alternative Options

Anterior cervical discectomy and fusion (ACDF) (A): Suitable for anteriorly located tumors or cervical disc disease.
Posterolateral approach (costotransversectomy) (C): Ideal for thoracic spine tumors.
Transoral odontoidectomy (D): Reserved for high cervical or odontoid tumors.

108
Q

A 2-year-old child presents with recurrent abdominal pain, vomiting and constipation. Diagnostic imaging reveals an intussusception. The best initial management is:
Options

A) Emergency laparotomy
B) Air enema reduction under fluoroscopy
C) IV fluids, antibiotics and observation
D) Gastrografin enema

A

Answer and Explanation

Correct answer: B) Air enema reduction under fluoroscopy
Explanation: For pediatric intussusception, air enema reduction under fluoroscopy is the preferred initial management.
Rationale

High success rate: 80-90% reduction success.
Minimally invasive: Avoids surgery.
Quick recovery: Reduces hospital stay.
Indications

Symptomatic intussusception: Abdominal pain, vomiting, bloody stools.
Diagnostic confirmation: Ultrasound or CT scan.
Stable patient: No signs of shock, peritonitis or free fluid.
Contraindications

Shock or instability: Requires surgical intervention.
Perforation or peritonitis: Surgical emergency.
Failed reduction: Requires surgical exploration.
Additional Considerations

Monitoring: Post-reduction observation for recurrence or complications.
Surgical backup: Preparedness for emergency laparotomy.
Investigation for lead point: Evaluate for underlying conditions (e.g., Meckel’s diverticulum).

109
Q

A 45-year-old woman presents with a 2-cm palpable breast lump. Imaging reveals a well-defined, BI-RADS 4 lesion. The appropriate next step is:
Options

A) Fine-needle aspiration cytology (FNAC)
B) Core needle biopsy (CNB)
C) Excisional biopsy
D) Mammography-guided vacuum-assisted biopsy

A

Correct answer: B) Core needle biopsy (CNB)
Explanation: Core needle biopsy (CNB) is the preferred diagnostic method for palpable breast lesions with a BI-RADS 4 classification.
Rationale

Tissue sampling: Provides histological diagnosis.
Accuracy: Higher than fine-needle aspiration cytology (FNAC).
Guides management: Informs treatment decisions.
Indications

Palpable lesions: Confirming diagnosis before surgery.
Imaging abnormalities: BI-RADS 4 or 5 lesions.
Preoperative diagnosis: Essential for breast cancer management.
Advantages Over Other Options

FNAC (A): Lower accuracy, limited tissue sampling.
Excisional biopsy (C): More invasive, often unnecessary.
Mammography-guided vacuum-assisted biopsy (D): Suitable for non-palpable, image-detected lesions.
Additional Considerations

Multidisciplinary team discussion: Radiologist, pathologist and surgeon collaboration.
Genetic testing: Consider for high-risk patients.
Preoperative axillary staging: Evaluate lymph node involvement.

110
Q

A 60-year-old male presents with painful urination, frequency and urgency. Urinalysis shows microscopic hematuria and positive leukocyte esterase. The likely diagnosis is:
Options

A) Prostate cancer
B) Bladder cancer
C) Urinary tract infection (UTI)
D) Benign prostatic hyperplasia (BPH)

A

Correct Answer

C) Urinary tract infection (UTI)
Explanation

Urinary tract infections (UTIs) are characterized by:
Dysuria: Painful urination
Frequency: Frequent voiding
Urgency: Sudden need to urinate
Hematuria: Blood in urine (microscopic or gross)
Leukocyte esterase: Positive test indicating white blood cells
Additional Considerations

Urinalysis confirmation: Verify infection.
Antibiotic therapy: Treat bacterial UTIs.
Further evaluation: Consider imaging or urology referral for recurrent or complex infections.
Other Options

Bladder cancer (B): Typically presents with gross hematuria.
Benign prostatic hyperplasia (D): Causes obstructive symptoms without infection signs.

111
Q

A 70-year-old male undergoes coronary artery bypass grafting (CABG). Postoperative echocardiography reveals severe left ventricular dysfunction. Management should focus on:
Options

A) Inotropic support
B) Mechanical circulatory support (MCS)
C) Heart transplantation evaluation
D) All of the above

A

Correct answer: D) All of the above
Explanation: Postoperative left ventricular dysfunction after CABG requires multifaceted management.
Initial Support

Inotropic support (A): Enhance contractility, maintain perfusion.
Vasopressors: Support blood pressure.
Advanced Therapies

Mechanical circulatory support (B): Consider intra-aortic balloon pump (IABP), extracorporeal membrane oxygenation (ECMO) or ventricular assist devices (VADs).
Heart transplantation evaluation (C): For severe, irreversible dysfunction.
Additional Measures

Optimize volume status: Ensure adequate filling pressures.
Electrolyte and acid-base balance: Correct abnormalities.
Reduce afterload: Vasodilators or ACE inhibitors.
Anti-arrhythmic prophylaxis: Prevent arrhythmias.
Early mobilization and rehabilitation: Enhance recovery.
Timing and Teamwork

Early intervention: Prevents further decline.
Multidisciplinary team: Collaborate with cardiologists, intensivists and surgeons.

112
Q

A 30-year-old presents with sudden, severe headache, vomiting and altered consciousness. Non-contrast CT scan shows subarachnoid hemorrhage. The most likely cause is:
Options

A) Aneurysmal rupture
B) Arteriovenous malformation (AVM)
C) Hypertensive hemorrhage
D) Traumatic brain injury

A

Correct Answer

A) Aneurysmal rupture
Explanation

Aneurysmal rupture is the most common cause (80-90%) of spontaneous subarachnoid hemorrhage.
Characteristics

Sudden, severe headache: “Thunderclap headache”
Vomiting and altered consciousness: Indicates increased intracranial pressure
Subarachnoid hemorrhage on CT: Blood in cerebrospinal fluid spaces
Risk Factors

Hypertension: Accelerates aneurysm formation
Family history: First-degree relatives
Smoking: Increases rupture risk
Immediate Actions

Stabilization: Secure airway, breathing and circulation
Neurosurgical consultation: Consider clipping or coiling
Monitoring: Watch for vasospasm, hydrocephalus and rebleeding
Other Options

Arteriovenous malformation (B): Less common, often congenital.
Hypertensive hemorrhage (C): Typically intraparenchymal, not subarachnoid.
Traumatic brain injury (D): History of trauma, different imaging findings.

113
Q

A 40-year-old male presents with abdominal pain, diarrhea and weight loss. Endoscopy reveals multiple duodenal ulcers. The most likely diagnosis is:
Options

A) Zollinger-Ellison syndrome
B) Peptic ulcer disease
C) Crohn’s disease
D) Giardiasis

A

Correct answer: A) Zollinger-Ellison syndrome
Explanation

Zollinger-Ellison syndrome (ZES) is characterized by:
Multiple duodenal ulcers: Unusual locations, large or recurrent.
Severe abdominal pain: Epigastric, radiating to back.
Diarrhea: Gastric acid hypersecretion.
Weight loss: Malabsorption.
Diagnostic Features

Elevated gastrin levels: Fasting gastrin >1000 pg/mL.
Gastric acid hypersecretion: Basal acid output increased.
Endoscopic findings: Multiple, large or atypical ulcers.
Management

Proton pump inhibitors (PPIs): Control acid secretion.
Surgical resection: Gastrinoma localization and removal.
Somatostatin analogs: Octreotide for symptom control.
Differential Diagnosis

Peptic ulcer disease (B): Typically single, benign ulcers.
Crohn’s disease (C): Inflammation, skip lesions, and fistulae.
Giardiasis (D): Parasitic infection causing diarrhea.
Key Considerations

Gastrinoma localization: Imaging studies (CT, MRI, PET).
Multiple endocrine neoplasia type 1 (MEN1): Association with ZES.
Long-term follow-up: Monitoring for recurrence, metastasis.

114
Q

A 25-year-old athlete presents with acute knee pain and instability after a twisting injury. Examination reveals a positive Lachman test. The most likely diagnosis is:
Options

A) Anterior cruciate ligament (ACL) tear
B) Posterior cruciate ligament (PCL) tear
C) Medial collateral ligament (MCL) tear
D) Meniscal tear

A

Correct answer: A) Anterior cruciate ligament (ACL) tear
Explanation

ACL tears typically result from twisting injuries, sudden stops or deceleration.
Diagnostic Features

Positive Lachman test: Excessive anterior tibial translation.
Anterior drawer test: Abnormal tibial movement.
Pivot shift test: Abnormal knee movement.
Knee instability: Giving way or buckling.
MRI confirmation: Definitive diagnosis.
Mechanism of Injury

Non-contact injuries: Sudden changes in direction.
Landing from jumps: Incorrect landing technique.
Deceleration: Quick reductions in speed.
Treatment Options

Conservative management: Rehabilitation for partial tears or low-activity patients.
Surgical reconstruction: ACL reconstruction for active patients or complete tears.
Physical therapy: Strengthening surrounding muscles.
Differential Diagnosis

Posterior cruciate ligament (PCL) tear (B): Posterior instability.
Medial collateral ligament (MCL) tear (C): Medial knee pain.
Meniscal tear (D): Joint line pain, locking or catching.
Key Considerations

Early diagnosis: Prevents secondary injuries.
Individualized treatment: Activity level, tear severity.
Rehabilitation focus: Strength, proprioception and agility.

115
Q

A 40-year-old presents with recurrent sore throats, fever and swollen tonsils. Tonsillectomy is considered. Preoperative evaluation should include:
Options

A) Complete blood count (CBC)
B) Bleeding disorders screening
C) Liver function tests (LFTs)
D) All of the above

A

Correct answer: D) All of the above
Preoperative Evaluation for Tonsillectomy

Complete Blood Count (CBC) (A): Assess for anemia, infection or bleeding disorders.
Bleeding Disorders Screening (B): Identify coagulopathy, von Willebrand disease or platelet dysfunction.
Liver Function Tests (LFTs) (C): Evaluate liver status, affecting metabolism of anesthetics and medications.
Additional Preoperative Considerations:
Medical history: Review for diabetes, hypertension or respiratory issues.
Allergy history: Document antibiotic allergies.
Current medications: Note anticoagulants, antiplatelets or immunosuppressants.
Physical examination: Assess airway, neck mobility and overall health.
Informed consent: Discuss risks, benefits and alternatives.
Importance of Comprehensive Evaluation

Risk stratification: Identify potential complications.
Perioperative planning: Optimize patient condition.
Anesthetic strategy: Tailor to patient needs.

116
Q

A 5-year-old presents with exertional dyspnea, fatigue and palpitations. Echocardiogram shows a 2:1 atrioventricular (AV) block. The likely diagnosis is:
Options

A) Complete heart block
B) Wolff-Parkinson-White syndrome
C) Atrioventricular canal defect
D) Ebstein’s anomaly

A

Correct Answer

A) Complete heart block
Explanation

Complete heart block (CHB) is characterized by:
Atrioventricular (AV) block: 2:1 or 3:1 block.
Exertional dyspnea: Reduced cardiac output.
Fatigue: Decreased exercise tolerance.
Palpitations: Awareness of abnormal heart rhythm.
Diagnostic Features

Electrocardiogram (ECG): AV block, widened QRS.
Echocardiogram: Confirm AV block, rule out structural anomalies.
Holter monitor: Document arrhythmias.
Management

Pacemaker implantation: Corrects AV block.
Medical therapy: Symptomatic relief.
Differential Diagnosis

Wolff-Parkinson-White (WPW) syndrome (B): Pre-excitation, tachycardia.
Atrioventricular canal defect (C): Congenital heart defect, heart failure.
Ebstein’s anomaly (D): Congenital tricuspid valve anomaly.
Key Considerations

Early diagnosis: Prevents complications.
Pacemaker timing: Based on symptoms, age and block severity.
Long-term follow-up: Monitor pacemaker function.

117
Q

A 50-year-old presents with progressive hemiparesis, seizures and cognitive decline. MRI reveals a left frontal lobe mass. The most likely diagnosis is:
Options

A) Glioblastoma multiforme
B) Meningioma
C) Brain metastasis
D) Low-grade glioma

A

Answer and Explanation

Correct answer: A) Glioblastoma multiforme
Explanation

Glioblastoma multiforme (GBM) is the most aggressive primary brain tumor.
Clinical Features

Progressive hemiparesis: Weakness on one side.
Seizures: Focal or generalized.
Cognitive decline: Memory, language or personality changes.
Headache and nausea: Increased intracranial pressure.
Diagnostic Imaging

MRI: Heterogeneous, contrast-enhancing mass.
CT scan: Hyperdense or isodense lesion.
Histopathology

High-grade astrocytoma: Microscopic examination.
Necrosis and angiogenesis: Hallmark features.
Management

Surgical resection: Maximal safe removal.
Radiation therapy: Adjuvant treatment.
Chemotherapy: Temozolomide.
Supportive care: Seizure control, symptom management.
Differential Diagnosis

Meningioma (B): Extra-axial, well-circumscribed.
Brain metastasis (C): Multiple lesions, known primary cancer.
Low-grade glioma (D): Slower progression, better prognosis.
Key Considerations

Multidisciplinary team: Neurosurgeon, oncologist, radiologist collaboration.
Surgical planning: Functional MRI, neuronavigation.
Prognosis: Generally poor, 5-year survival <10%.

118
Q

A 30-year-old traveler returns from Africa with fever, chills and flu-like symptoms. Laboratory results show thrombocytopenia and elevated liver enzymes. The most likely diagnosis is:
Options

A) Malaria
B) Typhoid fever
C) Yellow fever
D) Dengue fever

A

Correct answer: A) Malaria
Explanation

Malaria is a mosquito-borne disease prevalent in Africa, characterized by:
Clinical Features

Fever: Recurring, cyclical.
Chills: Rigors, sweating.
Flu-like symptoms: Headache, fatigue, muscle aches.
Thrombocytopenia: Low platelet count.
Elevated liver enzymes: Hepatocellular damage.
Diagnostic Tests

Peripheral smear: Parasites in red blood cells.
Rapid diagnostic tests (RDTs): Antigen detection.
PCR: Molecular confirmation.
Management

Artemisinin-based combination therapy (ACT): First-line treatment.
Supportive care: Fluids, antipyretics.
Hospitalization: Severe cases, monitoring.
Differential Diagnosis

Typhoid fever (B): Salmonella Typhi infection, abdominal symptoms.
Yellow fever (C): Hemorrhagic fever, jaundice.
Dengue fever (D): Severe flu-like illness, hemorrhagic manifestations.
Key Considerations

Prompt diagnosis: Prevents complications.
Chemoprophylaxis: For travelers to endemic areas.
Vector control: Mosquito elimination.